Menu Top
NCERT Exemplar Solutions
6th 7th 8th 9th 10th 11th 12th

Class 11th Chapters
1. Sets 2. Relations And Functions 3. Trigonometric Functions
4. Principle Of Mathematical Induction 5. Complex Numbers And Quadratic Equations 6. Linear Inequalities
7. Permutations And Combinations 8. Binomial Theorem 9. Sequence And Series
10. Straight Lines 11. Conic Sections 12. Introduction To Three Dimensional Geometry
13. Limits And Derivatives 14. Mathematical Reasoning 15. Statistics
16. Probability

Content On This Page
Examples
Example 1 to 7 (Short Answer Type Questions) Example 8 (Long Answer Type Questions) Example 9 to 15 (Multiple Choice Questions)
Exercise
Question 1 to 11 (Short Answer Type Questions) Question 12 to 17 (Long Answer Type Questions) Question 18 to 29 (Multiple Choice Questions)
Question 30 to 36 (True or False) Question 37 to 41 (Fill in the Blanks) Question 42 to 43 (Match the Following)


Chapter 16 Probability

Welcome to this essential resource providing comprehensive, step-by-step solutions for the Class 11 NCERT Exemplar problems focusing on the fundamental concepts of Probability. This chapter introduces the modern, rigorous framework for quantifying uncertainty, moving beyond intuitive notions towards a mathematically sound structure. The Exemplar questions are specifically designed to challenge students significantly, demanding a deeper understanding of the axiomatic approach, skillful application of set theory principles within probabilistic contexts, and proficiency in handling complex counting scenarios that often underpin probability calculations. Mastering these problems is crucial for building a solid foundation required in fields like statistics, data science, risk management, and theoretical computer science.

These solutions meticulously guide students through the foundational concepts, starting with the precise definition of a Sample Space ($S$) – the set of all possible outcomes of a random experiment. We explore how to define sample spaces for various experiments, including those involving multiple stages or complex selections, as presented in the Exemplar. Events ($E$) are rigorously treated as subsets of the sample space ($E \subseteq S$), and concepts like mutually exclusive events (disjoint sets, $E \cap F = \emptyset$) and exhaustive events (whose union covers the entire sample space) are clearly explained and applied. The transition from intuitive probability to the formal Axiomatic Approach (developed by Kolmogorov) is central. Our solutions emphasize the three core axioms:

Building upon these axioms, the solutions demonstrate the derivation and application of crucial probability theorems. These include $P(\emptyset) = 0$, the rule for complementary events $P(E') = 1 - P(E)$ (where $E'$ is 'not E'), the upper bound $P(E) \le 1$, and the highly important Addition Theorem for any two events: $P(E \cup F) = P(E) + P(F) - P(E \cap F)$. The extension of this theorem to three events is also covered, providing tools to tackle more complex 'OR' scenarios. A significant portion of probability calculations, especially in introductory contexts, relies on the assumption of equally likely outcomes. The solutions extensively cover the application of the formula $P(E) = \frac{\text{Number of outcomes favorable to E}}{\text{Total number of possible outcomes}} = \frac{n(E)}{n(S)}$.

The true challenge often lies in accurately determining $n(E)$ and $n(S)$, especially for the intricate experiments featured in the NCERT Exemplar. These problems frequently necessitate the sophisticated use of counting techniques learned in the Permutations and Combinations chapter. The solutions demonstrate how to apply formulas like $\binom{n}{r}$ (or $^nC_r$) and $P(n,r)$ (or $^nP_r$) correctly to count possibilities in scenarios involving drawing multiple cards from a deck under specific conditions, selecting items from groups, arranging objects, or analyzing multiple dice rolls. Careful translation of event descriptions – such as 'at least one', 'exactly one', 'A or B' ($A \cup B$), 'A and B' ($A \cap B$), 'neither A nor B' ($(A \cup B)'$) – into precise set operations and applying the relevant probability rules or theorems is a key skill highlighted. Addressing all Exemplar question formats (MCQs, Fill-in-the-Blanks, True/False, Short/Long Answer), these solutions provide the systematic framework, counting clarity, and theorem application mastery needed for success in advanced probability.



Solved Examples

Example 1 to 7 (Short Answer Type Questions)

Example 1: An ordinary deck of cards contains 52 cards divided into four suits. The red suits are diamonds and hearts and black suits are clubs and spades. The cards J, Q, and K are called face cards. Suppose we pick one card from the deck at random.

(a) What is the sample space of the experiment?

(b) What is the event that the chosen card is a black face card?

Answer:

We are considering the experiment of picking one card at random from a standard deck of 52 playing cards.


(a) Sample Space

The sample space, denoted by $S$, is the set of all possible outcomes of the experiment. When picking one card from a deck of 52 cards, each card is a distinct outcome.

The deck contains 4 suits: Hearts (H), Diamonds (D), Clubs (C), and Spades (S).

Each suit has 13 ranks: Ace (A), 2, 3, 4, 5, 6, 7, 8, 9, 10, Jack (J), Queen (Q), and King (K).

The sample space $S$ is the set of all possible combinations of rank and suit.

$S = \{ AC, 2C, ..., 10C, JC, QC, KC, AD, 2D, ..., 10D, JD, QD, KD, AH, 2H, ..., 10H, JH, QH, KH, AS, 2S, ..., 10S, JS, QS, KS \}

The total number of outcomes in the sample space is 52.


(b) Event

Let $E$ be the event that the chosen card is a black face card.

The black suits are Clubs (C) and Spades (S).

The face cards are Jack (J), Queen (Q), and King (K).

The black face cards are the cards from the black suits (Clubs and Spades) that are face cards (J, Q, K).

These cards are:

  • Jack of Clubs (JC)
  • Queen of Clubs (QC)
  • King of Clubs (KC)
  • Jack of Spades (JS)
  • Queen of Spades (QS)
  • King of Spades (KS)

The event $E$ is the set containing these outcomes:

$E = \{ JC, QC, KC, JS, QS, KS \}$

The number of outcomes in the event $E$ is 6.

Example 2: Suppose that each child born is equally likely to be a boy or a girl. Consider a family with exactly three children.

(a) List the eight elements in the sample space whose outcomes are all possible genders of the three children.

(b) Write each of the following events as a set and find its probability :

(i) The event that exactly one child is a girl.

(ii) The event that at least two children are girls

(iii) The event that no child is a girl

Answer:

The experiment involves considering the genders of exactly three children in a family, where each child is equally likely to be a boy (B) or a girl (G).


(a) Sample Space

The sample space, $S$, consists of all possible sequences of genders for the three children. Since there are two possible outcomes for each child and there are three children, the total number of outcomes is $2 \times 2 \times 2 = 8$.

Listing the possible outcomes:

  • BBB (Boy, Boy, Boy)
  • BBG (Boy, Boy, Girl)
  • BGB (Boy, Girl, Boy)
  • GBB (Girl, Boy, Boy)
  • BGG (Boy, Girl, Girl)
  • GBG (Girl, Boy, Girl)
  • GGB (Girl, Girl, Boy)
  • GGG (Girl, Girl, Girl)

Thus, the sample space is:

$S = \{ BBB, BBG, BGB, GBB, BGG, GBG, GGB, GGG \}$

The total number of outcomes in the sample space is $n(S) = 8$.


(b) Events and Probabilities

The probability of any event $E$ in this sample space is given by $P(E) = \frac{\text{Number of outcomes in } E}{\text{Total number of outcomes in } S} = \frac{n(E)}{n(S)}$. Since each outcome is equally likely, $P(\text{outcome}) = \frac{1}{8}$.


(i) The event that exactly one child is a girl.

Let $E_1$ be this event. The outcomes where exactly one child is a girl are those with one G and two Bs.

$E_1 = \{ BBG, BGB, GBB \}$

The number of outcomes in $E_1$ is $n(E_1) = 3$.

The probability of this event is:

$P(E_1) = \frac{n(E_1)}{n(S)} = \frac{3}{8}$


(ii) The event that at least two children are girls.

Let $E_2$ be this event. "At least two children are girls" means there are either exactly two girls or exactly three girls.

Outcomes with exactly two girls: BGG, GBG, GGB

Outcomes with exactly three girls: GGG

$E_2 = \{ BGG, GBG, GGB, GGG \}$

The number of outcomes in $E_2$ is $n(E_2) = 4$.

The probability of this event is:

$P(E_2) = \frac{n(E_2)}{n(S)} = \frac{4}{8} = \frac{1}{2}$


(iii) The event that no child is a girl.

Let $E_3$ be this event. "No child is a girl" means all three children are boys.

$E_3 = \{ BBB \}$

The number of outcomes in $E_3$ is $n(E_3) = 1$.

The probability of this event is:

$P(E_3) = \frac{n(E_3)}{n(S)} = \frac{1}{8}$

Example 3:

(a) How many two-digit positive integers are multiples of 3?

(b) What is the probability that a randomly chosen two-digit positive integer is a multiple of 3?

Answer:

We are dealing with two-digit positive integers and their properties as multiples of 3.


(a) Number of two-digit positive integers that are multiples of 3.

A two-digit positive integer is an integer $n$ such that $10 \leq n \leq 99$.

We need to find how many integers in this range are multiples of 3. These are numbers that can be written in the form $3k$ for some integer $k$.

The smallest two-digit multiple of 3 is $3 \times 4 = 12$.

The largest two-digit multiple of 3 is $3 \times 33 = 99$.

So, the two-digit multiples of 3 are $12, 15, 18, \ldots, 99$.

This sequence is an arithmetic progression with first term $a = 12$, common difference $d = 3$, and the last term $l = 99$.

Let the last term be the $n$-th term. Using the formula $l = a + (n-1)d$:

$99 = 12 + (n-1)3$

... (i)

Subtract 12 from both sides:

$99 - 12 = (n-1)3$

$87 = 3(n-1)$

Divide by 3:

$\frac{87}{3} = n-1$

$29 = n-1$

Add 1 to both sides:

$n = 29 + 1$

$n = 30$

Alternatively, we can find the number of multiples of 3 up to 99 and subtract the number of multiples of 3 up to 9.

Number of multiples of 3 up to 99 is $\lfloor \frac{99}{3} \rfloor = 33$.

Number of multiples of 3 up to 9 is $\lfloor \frac{9}{3} \rfloor = 3$.

Number of two-digit multiples of 3 = (Number of multiples of 3 up to 99) - (Number of multiples of 3 up to 9)

$33 - 3 = 30$

So, there are 30 two-digit positive integers that are multiples of 3.


(b) Probability that a randomly chosen two-digit positive integer is a multiple of 3.

The sample space for this experiment is the set of all two-digit positive integers.

The smallest two-digit integer is 10.

The largest two-digit integer is 99.

The total number of two-digit integers is the count from 10 to 99, inclusive.

Total number of integers = $99 - 10 + 1 = 90$.

So, the total number of outcomes in the sample space is $n(S) = 90$.

Let $E$ be the event that the randomly chosen two-digit positive integer is a multiple of 3.

From part (a), the number of two-digit positive integers that are multiples of 3 is $n(E) = 30$.

The probability of event $E$ is given by:

$P(E) = \frac{\text{Number of favourable outcomes}}{\text{Total number of outcomes}} = \frac{n(E)}{n(S)}$

$P(E) = \frac{30}{90}$

Simplifying the fraction:

$P(E) = \frac{\cancel{30}^{1}}{\cancel{90}_{3}}$

$P(E) = \frac{1}{3}$

The probability that a randomly chosen two-digit positive integer is a multiple of 3 is $\frac{1}{3}$.

Example 4: A typical PIN (personal identification number) is a sequence of any four symbols chosen from the 26 letters in the alphabet and the ten digits. If all PINs are equally likely, what is the probability that a randomly chosen PIN contains a repeated symbol?

Answer:

The problem asks for the probability that a randomly chosen four-symbol PIN contains a repeated symbol.


First, we determine the total number of possible symbols available for each position in the PIN. The symbols can be any of the 26 letters in the alphabet or any of the 10 digits (0 through 9).

Total number of distinct symbols = $26 + 10 = 36$.


Sample Space

A PIN is a sequence of four symbols. Since the symbols can be repeated, the total number of possible PINs is the number of choices for the first symbol multiplied by the number of choices for the second, third, and fourth symbols.

Total number of possible PINs = $36 \times 36 \times 36 \times 36 = 36^4$.

$36^4 = 1,679,616$

This is the size of our sample space, $n(S) = 1,679,616$.


Event of Interest

Let $E$ be the event that a randomly chosen PIN contains a repeated symbol.

It is often easier to calculate the probability of the complement event, $E^c$, which is the event that the PIN contains no repeated symbols.

If a PIN has no repeated symbols, all four symbols must be distinct.


Complement Event

The event $E^c$ is that the four symbols in the PIN are all distinct.

To form a PIN with distinct symbols, we have:

  • 36 choices for the first symbol.
  • 35 choices for the second symbol (it must be different from the first).
  • 34 choices for the third symbol (it must be different from the first two).
  • 33 choices for the fourth symbol (it must be different from the first three).

The number of ways to choose and arrange 4 distinct symbols from 36 is given by the number of permutations of 36 items taken 4 at a time, $P(36, 4)$.

$n(E^c) = 36 \times 35 \times 34 \times 33$

$36 \times 35 \times 34 \times 33 = 1,413,720$

This is the number of outcomes in the complement event, $n(E^c) = 1,413,720$.


Probability of the Complement Event

The probability of the PIN containing no repeated symbols is:

$P(E^c) = \frac{n(E^c)}{n(S)}$

$P(E^c) = \frac{1,413,720}{1,679,616}$

This fraction can be simplified:

$P(E^c) = \frac{36 \times 35 \times 34 \times 33}{36 \times 36 \times 36 \times 36} = \frac{35 \times 34 \times 33}{36 \times 36 \times 36}$

$P(E^c) = \frac{35 \times 17 \times 11 \times 2 \times 3}{36 \times 36 \times 36} = \frac{35 \times 17 \times 11 \times 6}{36 \times 36 \times 36}$

$P(E^c) = \frac{35 \times 17 \times 11}{\cancel{36}_{6} \times 36 \times 36} = \frac{35 \times 17 \times 11}{6 \times 36 \times 36} = \frac{6545}{7776}$

$P(E^c) = \frac{6545}{7776}$.


Probability of the Event

The probability of the PIN containing a repeated symbol is $P(E) = 1 - P(E^c)$.

$P(E) = 1 - \frac{6545}{7776}$

$P(E) = \frac{7776}{7776} - \frac{6545}{7776} = \frac{7776 - 6545}{7776}$

$P(E) = \frac{1231}{7776}$

The fraction $\frac{1231}{7776}$ cannot be simplified further as 1231 is a prime number and 7776 is not divisible by 1231.

The probability that a randomly chosen PIN contains a repeated symbol is $\frac{1231}{7776}$.

Example 5: An experiment has four possible outcomes A, B, C and D, that are mutually exclusive. Explain why the following assignments of probabilities are not permissible:

(a) P(A) = 0.12

P(B) = 0.63

P(C) = 0.45

P(D) = -0.20

(b) $P(A) = \frac{9}{120}$

$P(B) = \frac{45}{120}$

$P(C) = \frac{27}{120}$

$P(D) = \frac{46}{120}$

Answer:

For any valid probability assignment for outcomes in a sample space, two basic axioms of probability must be satisfied:

Axiom 1: The probability of any event (or outcome) must be a non-negative real number between 0 and 1, inclusive. That is, for any event $E$, $0 \leq P(E) \leq 1$.

Axiom 2: The sum of the probabilities of all mutually exclusive and exhaustive outcomes in the sample space must be exactly 1. If $A, B, C, D$ are the only possible and mutually exclusive outcomes, then $P(A) + P(B) + P(C) + P(D) = 1$.


Let's examine the given probability assignments based on these axioms.


(a) Given Probabilities:

$P(A) = 0.12$

$P(B) = 0.63$

$P(C) = 0.45$

$P(D) = -0.20$

Checking Axiom 1:

$P(A) = 0.12$ is between 0 and 1.

$P(B) = 0.63$ is between 0 and 1.

$P(C) = 0.45$ is between 0 and 1.

$P(D) = -0.20$ is not between 0 and 1, as it is a negative value.

Since the probability of outcome D is negative, this assignment violates Axiom 1.

Checking the sum (Axiom 2):

$P(A) + P(B) + P(C) + P(D) = 0.12 + 0.63 + 0.45 + (-0.20) = 1.00$.

The sum of probabilities is 1, which satisfies Axiom 2. However, because Axiom 1 is violated ($P(D) < 0$), this assignment is not permissible.


(b) Given Probabilities:

$P(A) = \frac{9}{120}$

$P(B) = \frac{45}{120}$

$P(C) = \frac{27}{120}$

$P(D) = \frac{46}{120}$

Checking Axiom 1:

All probabilities $\frac{9}{120}, \frac{45}{120}, \frac{27}{120}, \frac{46}{120}$ are positive fractions less than 1 (e.g., $\frac{46}{120} < \frac{120}{120} = 1$). So, Axiom 1 is satisfied for individual probabilities.

Checking the sum (Axiom 2):

We calculate the sum of the probabilities:

$P(A) + P(B) + P(C) + P(D) = \frac{9}{120} + \frac{45}{120} + \frac{27}{120} + \frac{46}{120}$

$ = \frac{9 + 45 + 27 + 46}{120}$

$ = \frac{127}{120}$

The sum of the probabilities is $\frac{127}{120}$, which is greater than 1. This violates Axiom 2.

Since the sum of the probabilities of all possible outcomes is greater than 1, this assignment is not permissible.

Example 6: Probability that a truck stopped at a roadblock will have faulty brakes or badly worn tires are 0.23 and 0.24, respectively. Also, the probability is 0.38 that a truck stopped at the roadblock will have faulty brakes and/or badly working tires. What is the probability that a truck stopped at this roadblock will have faulty breaks as well as badly worn tires?

Answer:

Let $B$ be the event that a truck stopped at the roadblock has faulty brakes.

Let $T$ be the event that a truck stopped at the roadblock has badly worn tires.


We are given the following probabilities:

Probability of faulty brakes, $P(B) = 0.23$

Probability of badly worn tires, $P(T) = 0.24$

Probability of faulty brakes and/or badly worn tires, $P(B \cup T) = 0.38$


We need to find the probability that a truck has faulty brakes as well as badly worn tires. This corresponds to the probability of the intersection of the events $B$ and $T$, which is denoted by $P(B \cap T)$.

The relationship between the probability of the union and the probability of the intersection of two events is given by the formula:

$P(B \cup T) = P(B) + P(T) - P(B \cap T)$

We can rearrange this formula to solve for $P(B \cap T)$:

$P(B \cap T) = P(B) + P(T) - P(B \cup T)$


Now, substitute the given values into the formula:

$P(B \cap T) = 0.23 + 0.24 - 0.38$

Performing the calculation:

$P(B \cap T) = 0.47 - 0.38$

$P(B \cap T) = 0.09$

Thus, the probability that a truck stopped at this roadblock will have faulty brakes as well as badly worn tires is $0.09$.

Example 7: If a person visits his dentist, suppose the probability that he will have his teeth cleaned is 0.48, the probability that he will have a cavity filled is 0.25, the probability that he will have a tooth extracted is 0.20, the probability that he will have a teeth cleaned and a cavity filled is 0.09, the probability that he will have his teeth cleaned and a tooth extracted is 0.12, the probability that he will have a cavity filled and a tooth extracted is 0.07, and the probability that he will have his teeth cleaned, a cavity filled, and a tooth extracted is 0.03. What is the probability that a person visiting his dentist will have atleast one of these things done to him?

Answer:

Let the events be defined as follows:

C: The person has his teeth cleaned.

F: The person has a cavity filled.

E: The person has a tooth extracted.


Given Probabilities:

The probability of having teeth cleaned is $P(C) = 0.48$.

The probability of having a cavity filled is $P(F) = 0.25$.

The probability of having a tooth extracted is $P(E) = 0.20$.

The probability of having teeth cleaned and a cavity filled is $P(C \cap F) = 0.09$.

The probability of having teeth cleaned and a tooth extracted is $P(C \cap E) = 0.12$.

The probability of having a cavity filled and a tooth extracted is $P(F \cap E) = 0.07$.

The probability of having teeth cleaned, a cavity filled, and a tooth extracted is $P(C \cap F \cap E) = 0.03$.


To Find:

The probability that a person visiting his dentist will have at least one of these things done to him. This is the probability of the union of the three events, $P(C \cup F \cup E)$.


Formula:

For three events $C$, $F$, and $E$, the probability of their union is given by the inclusion-exclusion principle:

$P(C \cup F \cup E) = P(C) + P(F) + P(E) - P(C \cap F) - P(C \cap E) - P(F \cap E) + P(C \cap F \cap E)$


Calculation:

Substitute the given probability values into the formula:

$P(C \cup F \cup E) = 0.48 + 0.25 + 0.20 - 0.09 - 0.12 - 0.07 + 0.03$

Sum the probabilities of the individual events:

$0.48 + 0.25 + 0.20 = 0.93$

Sum the probabilities of the pairwise intersections:

$0.09 + 0.12 + 0.07 = 0.28$

Now substitute these sums back into the main formula:

$P(C \cup F \cup E) = 0.93 - 0.28 + 0.03$

Perform the subtraction and addition:

$0.93 - 0.28 = 0.65$

$0.65 + 0.03 = 0.68$

So, the probability of having at least one of these things done is $0.68$.


Answer:

The probability that a person visiting his dentist will have at least one of these things done to him is $0.68$.

Example 8 (Long Answer Type Questions)

Example 8: An urn contains twenty white slips of paper numbered from 1 through 20, ten red slips of paper numbered from 1 through 10, forty yellow slips of paper numbered from 1 through 40, and ten blue slips of paper numbered from 1 through 10. If these 80 slips of paper are thoroughly shuffled so that each slip has the same probability of being drawn. Find the probabilities of drawing a slip of paper that is

(a) blue or white

(b) numbered 1, 2, 3, 4 or 5

(c) red or yellow and numbered 1, 2, 3 or 4

(d) numbered 5, 15, 25, or 35;

(e) white and numbered higher than 12 or yellow and numbered higher than 26.

Answer:

Given:

Number of white slips (1-20): 20

Number of red slips (1-10): 10

Number of yellow slips (1-40): 40

Number of blue slips (1-10): 10

Total number of slips in the urn is $20 + 10 + 40 + 10 = 80$.

The total number of possible outcomes when drawing a single slip is 80.


Solution:

Let $N(E)$ denote the number of favorable outcomes for an event $E$, and $N(S)$ denote the total number of possible outcomes. The probability of event $E$ is given by $P(E) = \frac{N(E)}{N(S)}$. Here, $N(S) = 80$.

(a) Drawing a blue or white slip.

Let $B$ be the event of drawing a blue slip and $W$ be the event of drawing a white slip.

Number of blue slips, $N(B) = 10$.

Number of white slips, $N(W) = 20$.

Since the events are mutually exclusive (a slip cannot be both blue and white), the number of favorable outcomes for drawing a blue or white slip is $N(B \cup W) = N(B) + N(W)$.

$N(B \cup W) = 10 + 20 = 30$.

The probability of drawing a blue or white slip is $P(B \cup W) = \frac{N(B \cup W)}{N(S)} = \frac{30}{80}$.

Simplifying the fraction, $P(B \cup W) = \frac{3}{8}$.


(b) Drawing a slip numbered 1, 2, 3, 4 or 5.

We count the number of slips with these numbers from each color category:

White slips (1-20) numbered 1, 2, 3, 4, 5: 5 slips.

Red slips (1-10) numbered 1, 2, 3, 4, 5: 5 slips.

Yellow slips (1-40) numbered 1, 2, 3, 4, 5: 5 slips.

Blue slips (1-10) numbered 1, 2, 3, 4, 5: 5 slips.

Total number of slips numbered 1, 2, 3, 4 or 5 is $5 + 5 + 5 + 5 = 20$.

The number of favorable outcomes is $N(\text{numbered 1,2,3,4,5}) = 20$.

The probability of drawing a slip numbered 1, 2, 3, 4 or 5 is $\frac{20}{80}$.

Simplifying the fraction, the probability is $\frac{1}{4}$.


(c) Drawing a slip that is red or yellow and numbered 1, 2, 3 or 4.

This means the slip is either (red AND numbered 1, 2, 3 or 4) OR (yellow AND numbered 1, 2, 3 or 4).

Number of red slips numbered 1, 2, 3 or 4:

Red slips are numbered 1-10. The numbers 1, 2, 3, 4 are within this range. So there are 4 such slips.

Number of yellow slips numbered 1, 2, 3 or 4:

Yellow slips are numbered 1-40. The numbers 1, 2, 3, 4 are within this range. So there are 4 such slips.

The events (red AND numbered 1,2,3,4) and (yellow AND numbered 1,2,3,4) are mutually exclusive.

Total number of favorable outcomes is $4 + 4 = 8$.

The probability is $\frac{8}{80}$.

Simplifying the fraction, the probability is $\frac{1}{10}$.


(d) Drawing a slip numbered 5, 15, 25, or 35.

We count the number of slips with these specific numbers from each color category:

White slips (1-20) numbered 5, 15: 2 slips.

Red slips (1-10) numbered 5, 15: Only 5 is in range (1-10). So 1 slip.

Yellow slips (1-40) numbered 5, 15, 25, 35: All are in range (1-40). So 4 slips.

Blue slips (1-10) numbered 5, 15, 25, 35: Only 5 is in range (1-10). So 1 slip.

Total number of slips numbered 5, 15, 25, or 35 is $2 + 1 + 4 + 1 = 8$.

The number of favorable outcomes is $N(\text{numbered 5,15,25,35}) = 8$.

The probability of drawing a slip numbered 5, 15, 25, or 35 is $\frac{8}{80}$.

Simplifying the fraction, the probability is $\frac{1}{10}$.


(e) Drawing a slip that is white and numbered higher than 12 or yellow and numbered higher than 26.

Let $E_1$ be the event the slip is white and numbered higher than 12.

Let $E_2$ be the event the slip is yellow and numbered higher than 26.

Number of white slips numbered higher than 12:

White slips are numbered from 1 to 20. Numbers higher than 12 are 13, 14, ..., 20.

Number of such slips is $20 - 13 + 1 = 8$. So, $N(E_1) = 8$.

Number of yellow slips numbered higher than 26:

Yellow slips are numbered from 1 to 40. Numbers higher than 26 are 27, 28, ..., 40.

Number of such slips is $40 - 27 + 1 = 14$. So, $N(E_2) = 14$.

The events $E_1$ and $E_2$ are mutually exclusive (a slip cannot be both white and yellow).

The number of favorable outcomes for $E_1 \cup E_2$ is $N(E_1 \cup E_2) = N(E_1) + N(E_2)$.

$N(E_1 \cup E_2) = 8 + 14 = 22$.

The probability of drawing a slip that is white and numbered higher than 12 or yellow and numbered higher than 26 is $\frac{22}{80}$.

Simplifying the fraction, the probability is $\frac{11}{40}$.

Example 9 to 15 (Multiple Choice Questions)

Choose the correct answer from given four options in each of the Examples 9 to 15 (M.C.Q.).

Example 9: In a leap year the probability of having 53 Sundays or 53 Mondays is

(A) $\frac{2}{7}$

(B) $\frac{3}{7}$

(C) $\frac{4}{7}$

(D) $\frac{5}{7}$

Answer:

Solution:

A leap year has 366 days.

We can express 366 days in terms of weeks and remaining days:

$366 = 52 \times 7 + 2$

So, a leap year has 52 full weeks and 2 extra days.

The 52 full weeks guarantee 52 Sundays and 52 Mondays.

The probability of having 53 Sundays or 53 Mondays depends on the two extra days.

The two extra days can be any consecutive pair of days of the week. The possible pairs are:

(Sunday, Monday)

(Monday, Tuesday)

(Tuesday, Wednesday)

(Wednesday, Thursday)

(Thursday, Friday)

(Friday, Saturday)

(Saturday, Sunday)

There are 7 equally likely possibilities for the two extra days.

Total number of possible outcomes for the two extra days = 7.

We are interested in the event of having 53 Sundays or 53 Mondays.

This happens if the two extra days include a Sunday or a Monday.

Let $S$ be the event that the two extra days result in 53 Sundays (i.e., one of the extra days is Sunday).

Let $M$ be the event that the two extra days result in 53 Mondays (i.e., one of the extra days is Monday).

The pairs that result in 53 Sundays are (Sunday, Monday) and (Saturday, Sunday).

The pairs that result in 53 Mondays are (Sunday, Monday) and (Monday, Tuesday).

We are looking for the probability of $S \cup M$. Using the principle of inclusion-exclusion:

$P(S \cup M) = P(S) + P(M) - P(S \cap M)$

The pairs resulting in 53 Sundays are (Sunday, Monday) and (Saturday, Sunday). $N(S) = 2$.

The pairs resulting in 53 Mondays are (Sunday, Monday) and (Monday, Tuesday). $N(M) = 2$.

The pair resulting in both 53 Sundays and 53 Mondays (53 of each) is (Sunday, Monday). $N(S \cap M) = 1$.

The number of favorable outcomes for having 53 Sundays or 53 Mondays is $N(S \cup M) = N(S) + N(M) - N(S \cap M) = 2 + 2 - 1 = 3$.

Alternatively, we can directly list the favorable outcomes for having 53 Sundays or 53 Mondays from the 7 pairs:

(Sunday, Monday) - Results in 53 Sundays and 53 Mondays.

(Monday, Tuesday) - Results in 53 Mondays.

(Saturday, Sunday) - Results in 53 Sundays.

The favorable pairs are (Sunday, Monday), (Monday, Tuesday), and (Saturday, Sunday).

Number of favorable outcomes = 3.

Total number of outcomes = 7.

The probability is $\frac{\text{Number of favorable outcomes}}{\text{Total number of outcomes}} = \frac{3}{7}$.


The correct answer is (B).

Example 10: Three digit numbers are formed using the digits 0, 2, 4, 6, 8. A number is chosen at random out of these numbers. What is the probability that this number has the same digits?

(A) $\frac{1}{16}$

(B) $\frac{16}{25}$

(C) $\frac{1}{645}$

(D) $\frac{1}{25}$

Answer:

Solution:

The digits available are 0, 2, 4, 6, 8 (a total of 5 digits).

We are forming three-digit numbers using these digits.

For a number to be a three-digit number, the first digit cannot be 0.

Let the three-digit number be represented as $d_1 d_2 d_3$, where $d_1$ is the hundreds digit, $d_2$ is the tens digit, and $d_3$ is the units digit.

Possible options for $d_1$: 2, 4, 6, 8 (4 options).

Possible options for $d_2$: 0, 2, 4, 6, 8 (5 options).

Possible options for $d_3$: 0, 2, 4, 6, 8 (5 options).

The total number of distinct three-digit numbers that can be formed is the product of the number of options for each digit.

Total number of possible outcomes, $N(S) = 4 \times 5 \times 5 = 100$.

Now, we need to find the number of outcomes where the number has the same digits.

This means $d_1 = d_2 = d_3$.

Since $d_1$ cannot be 0 (for a three-digit number), the common digit must be from the set $\{2, 4, 6, 8\}$.

If the common digit is 2, the number is 222.

If the common digit is 4, the number is 444.

If the common digit is 6, the number is 666.

If the common digit is 8, the number is 888.

These are the only three-digit numbers formed using the given digits that have the same digits.

The number of favorable outcomes, $N(E) = 4$.

The probability that a randomly chosen number has the same digits is given by:

$P(E) = \frac{N(E)}{N(S)}$

$P(E) = \frac{4}{100}$

Simplifying the fraction:

$P(E) = \frac{1}{25}$

The correct answer is (D).

Example 11: Three squares of chess board are selected at random. The probability of getting 2 squares of one colour and other of a different colour is

(A) $\frac{16}{21}$

(B) $\frac{8}{21}$

(C) $\frac{3}{32}$

(D) $\frac{3}{8}$

Answer:

Given:

A standard chessboard has $8 \times 8 = 64$ squares.

Number of white squares = 32.

Number of black squares = 32.

Three squares are selected at random.


Solution:

The total number of ways to select 3 squares from the 64 squares on the chessboard is given by the combination formula $\binom{n}{k} = \frac{n!}{k!(n-k)!}$.

Total number of possible outcomes, $N(S) = \binom{64}{3}$.

$N(S) = \frac{64 \times 63 \times 62}{3 \times 2 \times 1}$

$N(S) = 64 \times 21 \times 31$


We want to find the number of favorable outcomes, which is getting 2 squares of one colour and 1 square of a different colour.

This can happen in two mutually exclusive cases:

1. Selecting 2 white squares and 1 black square.

2. Selecting 2 black squares and 1 white square.

Number of ways to select 2 white squares from 32 is $\binom{32}{2}$.

Number of ways to select 1 black square from 32 is $\binom{32}{1}$.

Number of ways for Case 1 = $\binom{32}{2} \times \binom{32}{1}$.

Number of ways to select 2 black squares from 32 is $\binom{32}{2}$.

Number of ways to select 1 white square from 32 is $\binom{32}{1}$.

Number of ways for Case 2 = $\binom{32}{2} \times \binom{32}{1}$.

Total number of favorable outcomes, $N(E) = (\text{Number of ways for Case 1}) + (\text{Number of ways for Case 2})$.

$N(E) = \left(\binom{32}{2} \times \binom{32}{1}\right) + \left(\binom{32}{2} \times \binom{32}{1}\right)$

$N(E) = 2 \times \binom{32}{2} \times \binom{32}{1}$

Calculating the combinations:

$\binom{32}{2} = \frac{32 \times 31}{2} = 16 \times 31 = 496$.

$\binom{32}{1} = 32$.

$N(E) = 2 \times 496 \times 32$.


The probability $P(E)$ is the ratio of the number of favorable outcomes to the total number of possible outcomes.

$P(E) = \frac{N(E)}{N(S)} = \frac{2 \times \binom{32}{2} \times \binom{32}{1}}{\binom{64}{3}}$

Substitute the calculated combination values or the expanded forms:

$P(E) = \frac{2 \times \left(\frac{32 \times 31}{2}\right) \times 32}{\frac{64 \times 63 \times 62}{6}}$

$P(E) = \frac{32 \times 31 \times 32}{\frac{64 \times 63 \times 62}{6}}$

$P(E) = \frac{32 \times 31 \times 32 \times 6}{64 \times 63 \times 62}$

Simplifying the expression:

$P(E) = \frac{\cancel{32} \times \cancel{31} \times 32 \times 6}{(2 \times \cancel{32}) \times (63) \times (2 \times \cancel{31})}$

$P(E) = \frac{32 \times 6}{2 \times 63 \times 2}$

$P(E) = \frac{32 \times \cancel{6}^{3}}{4 \times 63}$

$P(E) = \frac{\cancel{32}^{8} \times 3}{\cancel{4}_{1} \times 63}$

$P(E) = \frac{8 \times 3}{63}$

$P(E) = \frac{24}{63}$

Divide numerator and denominator by 3:

$P(E) = \frac{\cancel{24}^{8}}{\cancel{63}^{21}}$

$P(E) = \frac{8}{21}$


Let's re-simplify the step $\frac{32 \times 32}{64 \times 21}$:

$P(E) = \frac{32 \times 32}{64 \times 21} = \frac{\cancel{32} \times 32}{(2 \times \cancel{32}) \times 21} = \frac{32}{2 \times 21} = \frac{\cancel{32}^{16}}{\cancel{2}_{1} \times 21} = \frac{16}{21}$.

My manual simplification steps were getting mixed up. The fraction $\frac{32 \times 31 \times 32 \times 6}{64 \times 63 \times 62}$ simplifies correctly as follows:

$P(E) = \frac{32 \times 31 \times 32 \times 6}{(2 \times 32) \times (21 \times 3) \times (2 \times 31)}$

$P(E) = \frac{\cancel{32} \times \cancel{31} \times 32 \times (2 \times 3)}{(2 \times \cancel{32}) \times (21 \times \cancel{3}) \times (2 \times \cancel{31})}$

$P(E) = \frac{32 \times 2}{2 \times 21 \times 2}$

$P(E) = \frac{\cancel{32}^{16} \times \cancel{2}}{\cancel{2} \times 21 \times \cancel{2}_{1}}$

$P(E) = \frac{16}{21}$


The probability of getting 2 squares of one colour and other of a different colour is $\frac{16}{21}$.

The correct answer is (A).

Example 12: If A and B are any two events having P (A ∪ B) = $\frac{1}{2}$ and $P (\overline{A})$ = $\frac{2}{3}$, then the probability of $\overline{A}$ ∩ B is

(A) $\frac{1}{2}$

(B) $\frac{2}{3}$

(C) $\frac{1}{6}$

(D) $\frac{1}{3}$

Answer:

Given:

$P(A \cup B) = \frac{1}{2}$

$P(\overline{A}) = \frac{2}{3}$


Solution:

We know that the probability of an event A and its complement $\overline{A}$ is related by the formula $P(A) + P(\overline{A}) = 1$.

So, we can find the probability of event A:

$P(A) = 1 - P(\overline{A})$

$P(A) = 1 - \frac{2}{3} = \frac{3}{3} - \frac{2}{3} = \frac{3-2}{3} = \frac{1}{3}$

We are asked to find the probability of $\overline{A} \cap B$. This represents the event where B occurs but A does not occur.

We can use the formula for the union of two events: $P(A \cup B) = P(A) + P(B) - P(A \cap B)$.

Also, we know that the event B can be partitioned into two mutually exclusive events: $A \cap B$ (A and B occur) and $\overline{A} \cap B$ (B occurs but A does not). So, $P(B) = P(A \cap B) + P(\overline{A} \cap B)$.

Rearranging this, we get $P(\overline{A} \cap B) = P(B) - P(A \cap B)$.

From the union formula, $P(A \cap B) = P(A) + P(B) - P(A \cup B)$.

Substitute this into the expression for $P(\overline{A} \cap B)$:

$P(\overline{A} \cap B) = P(B) - (P(A) + P(B) - P(A \cup B))$

$P(\overline{A} \cap B) = P(B) - P(A) - P(B) + P(A \cup B)$

$P(\overline{A} \cap B) = P(A \cup B) - P(A)$

Now, substitute the given values for $P(A \cup B)$ and the calculated value for $P(A)$:

$P(\overline{A} \cap B) = \frac{1}{2} - \frac{1}{3}$

To subtract the fractions, find a common denominator, which is 6.

$P(\overline{A} \cap B) = \frac{1 \times 3}{2 \times 3} - \frac{1 \times 2}{3 \times 2}$

$P(\overline{A} \cap B) = \frac{3}{6} - \frac{2}{6}$

$P(\overline{A} \cap B) = \frac{3 - 2}{6}$

$P(\overline{A} \cap B) = \frac{1}{6}$


The probability of $\overline{A} \cap B$ is $\frac{1}{6}$.

The correct answer is (C).

Example 13: Three of the six vertices of a regular hexagon are chosen at random. What is the probability that the triangle with these vertices is equilateral?

(A) $\frac{3}{10}$

(B) $\frac{3}{20}$

(C) $\frac{1}{20}$

(D) $\frac{1}{10}$

Answer:

Given:

A regular hexagon with 6 vertices.

Three vertices are chosen at random.


Solution:

The total number of ways to choose 3 vertices from the 6 vertices of a regular hexagon is given by the combination formula $\binom{n}{k}$.

Total number of possible outcomes, $N(S) = \binom{6}{3}$.

$N(S) = \frac{6!}{3!(6-3)!} = \frac{6 \times 5 \times 4}{3 \times 2 \times 1} = 20$.


We want to find the number of ways to choose 3 vertices that form an equilateral triangle.

In a regular hexagon, an equilateral triangle can be formed by connecting vertices that are separated by one vertex.

Let the vertices be numbered 1, 2, 3, 4, 5, 6 in order.

The possible equilateral triangles are formed by vertices (1, 3, 5) and (2, 4, 6).

There are exactly 2 such sets of vertices that form equilateral triangles.

The number of favorable outcomes, $N(E) = 2$.


The probability that the triangle formed by the three chosen vertices is equilateral is given by:

$P(E) = \frac{N(E)}{N(S)}$

$P(E) = \frac{2}{20}$

Simplifying the fraction:

$P(E) = \frac{\cancel{2}^{1}}{\cancel{20}^{10}}$

$P(E) = \frac{1}{10}$


The probability of getting an equilateral triangle is $\frac{1}{10}$.

The correct answer is (D).

Example 14: If A, B, C are three mutually exclusive and exhaustive events of an experiment such that

3P(A) = 2P(B) = P(C), then P(A) is equal to

(A) $\frac{1}{11}$

(B) $\frac{2}{11}$

(C) $\frac{5}{11}$

(D) $\frac{6}{11}$

Answer:

Given:

A, B, and C are three mutually exclusive and exhaustive events.

$3P(A) = 2P(B) = P(C)$


Solution:

Since A, B, and C are mutually exclusive and exhaustive events, the sum of their probabilities is equal to the probability of the sample space, which is 1.

$P(A) + P(B) + P(C) = 1$

We are given the relationship between the probabilities:

$3P(A) = 2P(B) = P(C)$

From $3P(A) = 2P(B)$, we can express $P(B)$ in terms of $P(A)$:

$P(B) = \frac{3}{2}P(A)$

From $3P(A) = P(C)$, we can express $P(C)$ in terms of $P(A)$:

$P(C) = 3P(A)$

Substitute these expressions for $P(B)$ and $P(C)$ into the equation $P(A) + P(B) + P(C) = 1$:

$P(A) + \frac{3}{2}P(A) + 3P(A) = 1$

Combine the terms involving $P(A)$ by finding a common denominator (which is 2):

$(1 + \frac{3}{2} + 3) P(A) = 1$

$(\frac{2}{2} + \frac{3}{2} + \frac{6}{2}) P(A) = 1$

$(\frac{2 + 3 + 6}{2}) P(A) = 1$

$\frac{11}{2} P(A) = 1$

Solve for $P(A)$:

$P(A) = \frac{1}{\frac{11}{2}}$

$P(A) = 1 \times \frac{2}{11}$

$P(A) = \frac{2}{11}$


The probability of event A is $\frac{2}{11}$.

The correct answer is (B).

Example 15: One mapping (function) is selected at random from all the mappings of the set A = {1, 2, 3, ..., n} into itself. The probability that the mapping selected is one to one is

(A) $\frac{1}{n^n}$

(B) $\frac{1}{n!}$

(C) $\frac{(n − 1)!}{n^{n−1}}$

(D) none of these

Answer:

Solution:

Let the set A be given by $A = \{1, 2, 3, \dots, n\}$. The set A has $n$ elements.

We are considering mappings (functions) from the set A to itself, i.e., $f: A \to A$.

To find the total number of possible mappings from A to A, consider each element in the domain A. For each element in the domain, there are $n$ possible images in the codomain A.

For the first element (1), there are $n$ choices for its image.

For the second element (2), there are $n$ choices for its image.

...

For the $n$-th element ($n$), there are $n$ choices for its image.

The total number of possible mappings from A to A is the product of the number of choices for each element in the domain.

Total number of mappings = $\underbrace{n \times n \times \dots \times n}_{n \text{ times}} = n^n$.

Total number of possible outcomes, $N(S) = n^n$.


We want to find the number of mappings that are one-to-one (injective).

A one-to-one mapping $f: A \to A$ requires that if $x_1 \ne x_2$, then $f(x_1) \ne f(x_2)$ for all $x_1, x_2 \in A$. Since the domain and codomain are the same finite set A, a one-to-one mapping is also an onto mapping (surjective), and therefore a bijection (permutation).

To form a one-to-one mapping:

For the first element (1) in the domain, there are $n$ choices for its image in the codomain.

For the second element (2) in the domain, its image must be different from the image of 1. So, there are $n-1$ remaining choices for its image.

For the third element (3) in the domain, its image must be different from the images of 1 and 2. So, there are $n-2$ remaining choices for its image.

...

For the $n$-th element ($n$) in the domain, its image must be different from the images of all previous $n-1$ elements. So, there is only $n - (n-1) = 1$ remaining choice for its image.

The number of one-to-one mappings is the product of the number of choices for each element.

Number of one-to-one mappings = $n \times (n-1) \times (n-2) \times \dots \times 1 = n!$.

Number of favorable outcomes, $N(E) = n!$.


The probability that the selected mapping is one-to-one is the ratio of the number of one-to-one mappings to the total number of mappings.

$P(\text{one-to-one mapping}) = \frac{N(E)}{N(S)} = \frac{n!}{n^n}$.

Now, let's compare this result with the given options.

Option (A): $\frac{1}{n^n}$

Option (B): $\frac{1}{n!}$

Option (C): $\frac{(n − 1)!}{n^{n−1}}$

Let's simplify our result $\frac{n!}{n^n}$ and compare it with Option (C).

We know that $n! = n \times (n-1)!$.

We also know that $n^n = n \times n^{n-1}$.

So, $\frac{n!}{n^n} = \frac{n \times (n-1)!}{n \times n^{n-1}}$.

We can cancel out one factor of $n$ from the numerator and the denominator:

$\frac{\cancel{n} \times (n-1)!}{\cancel{n} \times n^{n-1}} = \frac{(n-1)!}{n^{n-1}}$.

Our calculated probability $\frac{n!}{n^n}$ is equal to the expression in Option (C).


The probability that the mapping selected is one-to-one is $\frac{(n − 1)!}{n^{n−1}}$.

The correct answer is (C).



Exercise

Question 1 to 11 (Short Answer Type Questions)

Question 1. If the letters of the word ALGORITHM are arranged at random in a row what is the probability the letters GOR must remain together as a unit?

Answer:

Solution:


The word ALGORITHM consists of 9 distinct letters: A, L, G, O, R, I, T, H, M.

Total Number of Arrangements:

The total number of ways to arrange 9 distinct letters in a row is given by the factorial of the number of letters.

Total number of arrangements $= 9!$

$9! = 9 \times 8 \times 7 \times 6 \times 5 \times 4 \times 3 \times 2 \times 1 = 362,880$


Favorable Arrangements (GOR together as a unit):

We want the letters G, O, R to remain together as a single unit. Treat the group (GOR) as one block.

Now, we are arranging the following 7 entities:

(GOR), A, L, I, T, H, M

The number of ways to arrange these 7 distinct entities is $7!$.

Within the unit (GOR), the letters G, O, and R can be arranged among themselves in $3!$ ways.

The possible arrangements of GOR within the unit are GOR, GRO, OGR, ORG, RGO, ROG.

Number of arrangements of the unit (GOR) $= 3! = 3 \times 2 \times 1 = 6$

The total number of favorable arrangements where GOR stay together as a unit is the product of the number of ways to arrange the 7 entities and the number of ways to arrange letters within the GOR unit.

Number of favorable arrangements $= (\text{Number of ways to arrange the 7 entities}) \times (\text{Number of ways to arrange GOR within the unit})$

Number of favorable arrangements $= 7! \times 3!$

$7! = 5,040$

$3! = 6$

Number of favorable arrangements $= 5,040 \times 6 = 30,240$


Probability:

The probability of an event is the ratio of the number of favorable outcomes to the total number of possible outcomes.

Probability (GOR together) = $\frac{\text{Number of favorable arrangements}}{\text{Total number of arrangements}}$

Probability (GOR together) $= \frac{7! \times 3!}{9!}$

We can simplify this expression:

Probability $= \frac{7! \times 3!}{9 \times 8 \times 7!}$

Cancel out $7!$ from the numerator and the denominator.

Probability $= \frac{3!}{9 \times 8}$

Substitute the value of $3!$:

$3! = 6$

Probability $= \frac{6}{72}$

Simplify the fraction by dividing both numerator and denominator by their greatest common divisor, which is 6.

Probability $= \frac{\cancel{6}^{1}}{\cancel{72}_{12}} = \frac{1}{12}$


The probability that the letters GOR must remain together as a unit when the letters of the word ALGORITHM are arranged at random in a row is $\frac{1}{12}$.

Question 2. Six new employees, two of whom are married to each other, are to be assigned six desks that are lined up in a row. If the assignment of employees to desks is made randomly, what is the probability that the married couple will have nonadjacent desks?

[Hint: First find the probability that the couple has adjacent desks, and then subtract it from 1.]

Answer:

Given:

6 new employees are to be assigned 6 desks that are lined up in a row.

Two of the employees form a married couple.


To Find:

The probability that the married couple will have nonadjacent desks when the assignment of employees to desks is made randomly.


Solution:

Let the six employees be $E_1, E_2, E_3, E_4, E_5, E_6$. Let $E_1$ and $E_2$ represent the married couple.

We are asked to find the probability that $E_1$ and $E_2$ are assigned nonadjacent desks.

Using the hint provided, we will first calculate the probability that the married couple is assigned adjacent desks, and then subtract this probability from 1.


Total Number of Arrangements:

The total number of ways to assign 6 distinct employees to 6 distinct desks in a row is the number of permutations of 6 items.

Total number of arrangements $= 6!$

$6! = 6 \times 5 \times 4 \times 3 \times 2 \times 1 = 720$


Number of Arrangements where the Married Couple is Adjacent:

To count the arrangements where the married couple is adjacent, we treat the couple ($E_1, E_2$) as a single unit.

Now we have 5 entities to arrange: the coupled unit and the remaining 4 employees ($E_3, E_4, E_5, E_6$).

The number of ways to arrange these 5 distinct entities is $5!$.

$5! = 5 \times 4 \times 3 \times 2 \times 1 = 120$

Within the couple unit ($E_1, E_2$), the two individuals can be arranged in $2!$ ways (either $E_1$ is to the left of $E_2$, or $E_2$ is to the left of $E_1$).

$2! = 2 \times 1 = 2$

The total number of arrangements where the married couple is adjacent is the product of the number of ways to arrange the 5 entities and the number of ways to arrange the couple within their unit.

Number of adjacent arrangements $= 5! \times 2! = 120 \times 2 = 240$


Probability that the Married Couple is Adjacent:

The probability that the married couple is adjacent is the ratio of the number of adjacent arrangements to the total number of arrangements.

Probability (Adjacent) = $\frac{\text{Number of adjacent arrangements}}{\text{Total number of arrangements}}$

Probability (Adjacent) $= \frac{5! \times 2!}{6!}$

Probability (Adjacent) $= \frac{240}{720}$

Simplifying the fraction:

Probability (Adjacent) $= \frac{\cancel{240}^{1}}{\cancel{720}_{3}} = \frac{1}{3}$


Probability that the Married Couple is Nonadjacent:

The event that the married couple is nonadjacent is the complement of the event that the married couple is adjacent.

Probability (Nonadjacent) = $1 - $ Probability (Adjacent)

Probability (Nonadjacent) $= 1 - \frac{1}{3}$

Probability (Nonadjacent) $= \frac{3}{3} - \frac{1}{3} = \frac{2}{3}$


The probability that the married couple will have nonadjacent desks is $\frac{2}{3}$.

Question 3. Suppose an integer from 1 through 1000 is chosen at random, find the probability that the integer is a multiple of 2 or a multiple of 9.

Answer:

Given:

An integer is chosen at random from 1 through 1000.


To Find:

The probability that the integer is a multiple of 2 or a multiple of 9.


Solution:

The total number of integers from 1 through 1000 is 1000.

Total number of possible outcomes = 1000.

Let A be the event that the integer is a multiple of 2.

Let B be the event that the integer is a multiple of 9.

We want to find the probability of the event A or B, which is $P(A \cup B)$.

Using the Principle of Inclusion-Exclusion:

$P(A \cup B) = P(A) + P(B) - P(A \cap B)$

First, find the number of integers that are multiples of 2 in the range [1, 1000].

Number of multiples of 2 = $\lfloor \frac{1000}{2} \rfloor = 500$

So, the number of outcomes in event A is 500.

Next, find the number of integers that are multiples of 9 in the range [1, 1000].

Number of multiples of 9 = $\lfloor \frac{1000}{9} \rfloor = 111$

So, the number of outcomes in event B is 111.

Now, find the number of integers that are multiples of both 2 and 9. An integer that is a multiple of both 2 and 9 must be a multiple of their least common multiple (LCM).

LCM(2, 9) = 18

So, we need to find the number of multiples of 18 in the range [1, 1000].

Number of multiples of 18 = $\lfloor \frac{1000}{18} \rfloor = 55$

So, the number of outcomes in event $A \cap B$ is 55.

Now, apply the Principle of Inclusion-Exclusion to find the number of integers that are multiples of 2 or 9.

Number of (multiples of 2 or 9) = (Number of multiples of 2) + (Number of multiples of 9) - (Number of multiples of 18)

Number of (multiples of 2 or 9) $= 500 + 111 - 55$

Number of (multiples of 2 or 9) $= 611 - 55 = 556$

This is the number of favorable outcomes.

The probability is the ratio of the number of favorable outcomes to the total number of possible outcomes.

Probability (multiple of 2 or 9) $= \frac{\text{Number of favorable outcomes}}{\text{Total number of possible outcomes}}$

Probability $= \frac{556}{1000}$

We can simplify this fraction by dividing the numerator and denominator by their greatest common divisor. Both are divisible by 4.

$556 \div 4 = 139$

$1000 \div 4 = 250$

Probability $= \frac{139}{250}$


The probability that the integer chosen is a multiple of 2 or a multiple of 9 is $\frac{139}{250}$.

Question 4. An experiment consists of rolling a die until a 2 appears.

(i) How many elements of the sample space correspond to the event that the 2 appears on the kth roll of the die?

(ii) How many elements of the sample space correspond to the event that the 2 appears not later than the kth roll of the die?

[Hint: (a) First (k – 1) rolls have 5 outcomes each and kth rolls should result in 1outcomes. (b) 1 + 5 + 52 + ... + 5k–1.]

Answer:

Solution:

The experiment involves rolling a standard six-sided die until a 2 appears for the first time. The possible outcomes on each roll are {1, 2, 3, 4, 5, 6}.


(i) Number of elements when 2 appears on the kth roll:

For the 2 to appear for the first time on the kth roll, the following conditions must be met:

The first $k-1$ rolls must not result in a 2. There are 5 possible outcomes for each of these rolls (any number except 2).

The kth roll must result in a 2. There is only 1 possible outcome for this roll (the number 2).

Since the rolls are independent events, the total number of elements in the sample space corresponding to this event is the product of the number of outcomes for each roll.

Number of elements = (Number of outcomes for roll 1) $\times$ (Number of outcomes for roll 2) $\times$ ... $\times$ (Number of outcomes for roll k-1) $\times$ (Number of outcome for roll k)

Number of elements = $5 \times 5 \times \dots \times 5$ (for the first $k-1$ rolls) $\times 1$ (for the kth roll)

Number of elements $= 5^{k-1} \times 1 = 5^{k-1}$

So, there are $5^{k-1}$ elements in the sample space where the 2 appears exactly on the kth roll for the first time.


(ii) Number of elements when 2 appears not later than the kth roll:

The event that the 2 appears not later than the kth roll means that the first 2 appears on the 1st roll OR the 2nd roll OR the 3rd roll OR ... OR the kth roll.

Let $E_i$ be the event that the 2 appears for the first time on the ith roll.

We want to find the number of elements in the event $E_1 \cup E_2 \cup \dots \cup E_k$.

Since these events ($E_i$) are mutually exclusive (the first 2 cannot appear for the first time on both the ith and jth roll if $i \neq j$), the number of elements in their union is the sum of the number of elements in each event.

Number of elements = Number of elements in $E_1$ + Number of elements in $E_2$ + ... + Number of elements in $E_k$

Using the result from part (i), the number of elements in $E_i$ is $5^{i-1}$.

Number of elements = $5^{1-1} + 5^{2-1} + 5^{3-1} + \dots + 5^{k-1}$

Number of elements = $5^0 + 5^1 + 5^2 + \dots + 5^{k-1}$

Number of elements = $1 + 5 + 5^2 + \dots + 5^{k-1}$

This is a finite geometric series with first term $a = 1$, common ratio $r = 5$, and number of terms $n = k$.

The sum of a finite geometric series is given by the formula $S_n = a \frac{r^n - 1}{r - 1}$.

In this case, $S_k = 1 \times \frac{5^k - 1}{5 - 1} = \frac{5^k - 1}{4}$.

So, there are $\frac{5^k - 1}{4}$ elements in the sample space where the 2 appears not later than the kth roll.

Question 5. A die is loaded in such a way that each odd number is twice as likely to occur as each even number. Find P(G), where G is the event that a number greater than 3 occurs on a single roll of the die.

Answer:

Given:

A loaded die where the probability of an odd number is twice the probability of an even number.

Event G is that a number greater than 3 occurs on a single roll.


To Find:

The probability of event G, P(G).


Solution:

The possible outcomes of a single roll of a standard die are {1, 2, 3, 4, 5, 6}.

Let $P(\text{even})$ be the probability of rolling an even number (2, 4, or 6).

Let $P(\text{odd})$ be the probability of rolling an odd number (1, 3, or 5).

According to the problem statement, $P(\text{odd}) = 2 \times P(\text{even})$.

Since each even number is equally likely, the probability of rolling a specific even number (2, 4, or 6) is the same. Let this probability be $p$.

So, $P(2) = P(4) = P(6) = p$.

Since each odd number is equally likely and its probability is twice that of an even number, the probability of rolling a specific odd number (1, 3, or 5) is $2p$.

So, $P(1) = P(3) = P(5) = 2p$.

The sum of probabilities of all possible outcomes must equal 1.

$P(1) + P(2) + P(3) + P(4) + P(5) + P(6) = 1$

$2p + p + 2p + p + 2p + p = 1$

Combine the terms with $p$:

$(2+1+2+1+2+1)p = 1$

$9p = 1$

Solve for $p$:

$p = \frac{1}{9}$

Now we know the probability of each outcome:

$P(2) = P(4) = P(6) = \frac{1}{9}$

$P(1) = P(3) = P(5) = 2 \times \frac{1}{9} = \frac{2}{9}$

The event G is that a number greater than 3 occurs. The outcomes in event G are {4, 5, 6}.

To find the probability of event G, we sum the probabilities of the outcomes in G.

$P(G) = P(4) + P(5) + P(6)$

$P(G) = \frac{1}{9} + \frac{2}{9} + \frac{1}{9}$

$P(G) = \frac{1+2+1}{9} = \frac{4}{9}$


The probability of event G is $\frac{4}{9}$.

Question 6. In a large metropolitan area, the probabilities are .87, .36, .30 that a family (randomly chosen for a sample survey) owns a colour television set, a black and white television set, or both kinds of sets. What is the probability that a family owns either anyone or both kinds of sets?

Answer:

Given:

Let C be the event that a family owns a colour television set.

Let B be the event that a family owns a black and white television set.

We are given the following probabilities:

$P(C) = 0.87$

$P(B) = 0.36$

The probability that a family owns both kinds of sets is given as:

$P(C \cap B) = 0.30$


To Find:

The probability that a family owns either one or both kinds of sets. This corresponds to the probability of the union of events C and B, denoted as $P(C \cup B)$.


Solution:

We use the formula for the probability of the union of two events, which is given by:

$P(C \cup B) = P(C) + P(B) - P(C \cap B)$

Substitute the given values into the formula:

$P(C \cup B) = 0.87 + 0.36 - 0.30$

First, add $P(C)$ and $P(B)$:

$0.87 + 0.36 = 1.23$

Now, subtract $P(C \cap B)$ from the result:

$1.23 - 0.30 = 0.93$

So, the probability that a family owns either a colour television set or a black and white television set or both is 0.93.


The probability that a family owns either anyone or both kinds of sets is $0.93$.

Question 7. If A and B are mutually exclusive events, P (A) = 0.35 and P (B) = 0.45, find

(a) P (A′)

(b) P (B′)

(c) P (A ∪ B)

(d) P (A ∩ B)

(e) P (A ∩ B′)

(f) P (A′ ∩ B′)

Answer:

Given:

A and B are mutually exclusive events.

$P(A) = 0.35$

$P(B) = 0.45$


To Find:

(a) $P(A')$

(b) $P(B')$

(c) $P(A \cup B)$

(d) $P(A \cap B)$

(e) $P(A \cap B')$

(f) $P(A' \cap B')$


Solution:

Since A and B are mutually exclusive events, they cannot occur simultaneously. This means the probability of their intersection is 0, i.e., $P(A \cap B) = 0$.


(a) $P(A')$

The probability of the complement of an event A is given by $P(A') = 1 - P(A)$.

$P(A') = 1 - 0.35$

$P(A') = 0.65$


(b) $P(B')$

The probability of the complement of an event B is given by $P(B') = 1 - P(B)$.

$P(B') = 1 - 0.45$

$P(B') = 0.55$


(c) $P(A \cup B)$

For any two events A and B, the probability of their union is $P(A \cup B) = P(A) + P(B) - P(A \cap B)$.

Since A and B are mutually exclusive, $P(A \cap B) = 0$.

Therefore, $P(A \cup B) = P(A) + P(B)$.

$P(A \cup B) = 0.35 + 0.45$

$P(A \cup B) = 0.80$


(d) $P(A \cap B)$

Since A and B are mutually exclusive events, their intersection is the empty set ($\emptyset$). The probability of an empty set is 0.

$P(A \cap B) = P(\emptyset)$

$P(A \cap B) = 0$


(e) $P(A \cap B')$

The intersection $A \cap B'$ represents the event where A occurs and B does not occur. Since A and B are mutually exclusive, if A occurs, B cannot occur. Therefore, the event A occurring implies that B must not occur, which means the outcome is in $B'$. Thus, the event $A \cap B'$ is the same as event A.

$A \cap B' = A$

So, $P(A \cap B') = P(A)$.

$P(A \cap B') = 0.35$


(f) $P(A' \cap B')$

The intersection $A' \cap B'$ represents the event where neither A occurs nor B occurs. Using De Morgan's Law, this is equivalent to the complement of the union of A and B.

$A' \cap B' = (A \cup B)'$

The probability of the complement of an event is $P(E') = 1 - P(E)$.

$P(A' \cap B') = P((A \cup B)')$

$P(A' \cap B') = 1 - P(A \cup B)$

From part (c), we found $P(A \cup B) = 0.80$.

$P(A' \cap B') = 1 - 0.80$

$P(A' \cap B') = 0.20$

Question 8. A team of medical students doing their internship have to assist during surgeries at a city hospital. The probabilities of surgeries rated as very complex, complex, routine, simple or very simple are respectively, 0.15, 0.20, 0.31, 0.26, .08. Find the probabilities that a particular surgery will be rated

(a) complex or very complex;

(b) neither very complex nor very simple;

(c) routine or complex

(d) routine or simple

Answer:

Given:

Let the events be:

VC: Surgery is rated Very Complex, $P(VC) = 0.15$

C: Surgery is rated Complex, $P(C) = 0.20$

R: Surgery is rated Routine, $P(R) = 0.31$

S: Surgery is rated Simple, $P(S) = 0.26$

VS: Surgery is rated Very Simple, $P(VS) = 0.08$

The categories (Very Complex, Complex, Routine, Simple, Very Simple) are mutually exclusive, meaning a surgery cannot belong to more than one category simultaneously.


To Find:

(a) $P(\text{Complex or Very Complex})$

(b) $P(\text{Neither Very Complex nor Very Simple})$

(c) $P(\text{Routine or Complex})$

(d) $P(\text{Routine or Simple})$


Solution:

Since the different ratings are mutually exclusive events, the probability of the union of any two (or more) ratings is simply the sum of their individual probabilities.


(a) Probability of being Complex or Very Complex:

This is the probability of the event C or VC, i.e., $P(C \cup VC)$.

$P(C \cup VC) = P(C) + P(VC)$ (Since C and VC are mutually exclusive)

$P(C \cup VC) = 0.20 + 0.15$

$P(C \cup VC) = 0.35$


(b) Probability of being Neither Very Complex nor Very Simple:

This means the surgery is not in the VC category and not in the VS category. This is the complement of the event (VC or VS). So we want $P((VC \cup VS)')$.

$P((VC \cup VS)') = 1 - P(VC \cup VS)$

Since VC and VS are mutually exclusive, $P(VC \cup VS) = P(VC) + P(VS)$.

$P(VC \cup VS) = 0.15 + 0.08 = 0.23$

$P((VC \cup VS)') = 1 - 0.23 = 0.77$

Alternatively, "neither Very Complex nor Very Simple" means the surgery is Complex, Routine, or Simple. Since these events are mutually exclusive:

$P(\text{C or R or S}) = P(C) + P(R) + P(S)$

$P(\text{C or R or S}) = 0.20 + 0.31 + 0.26 = 0.77$


(c) Probability of being Routine or Complex:

This is the probability of the event R or C, i.e., $P(R \cup C)$.

$P(R \cup C) = P(R) + P(C)$ (Since R and C are mutually exclusive)

$P(R \cup C) = 0.31 + 0.20$

$P(R \cup C) = 0.51$


(d) Probability of being Routine or Simple:

This is the probability of the event R or S, i.e., $P(R \cup S)$.

$P(R \cup S) = P(R) + P(S)$ (Since R and S are mutually exclusive)

$P(R \cup S) = 0.31 + 0.26$

$P(R \cup S) = 0.57$

Question 9. Four candidates A, B, C, D have applied for the assignment to coach a school cricket team. If A is twice as likely to be selected as B, and B and C are given about the same chance of being selected, while C is twice as likely to be selected as D, what are the probabilities that

(a) C will be selected?

(b) A will not be selected?

Answer:

Given:

Four candidates A, B, C, and D.

The probabilities of their selection are related as follows:

A is twice as likely as B: $P(A) = 2 P(B)$

B and C are given about the same chance: Assuming "about the same chance" means equal chance, $P(B) = P(C)$.

C is twice as likely as D: $P(C) = 2 P(D)$

Since A, B, C, and D are the only candidates, the sum of their probabilities of being selected must be 1.

$P(A) + P(B) + P(C) + P(D) = 1$


To Find:

(a) The probability that C will be selected, $P(C)$.

(b) The probability that A will not be selected, $P(A')$.


Solution:

Let's express the probabilities of A, B, and C in terms of the probability of D.

From the given information, we have:

$P(C) = 2 P(D)$

$P(B) = P(C)$

Substituting the first equation into the second:

$P(B) = 2 P(D)$

Also, we are given $P(A) = 2 P(B)$.

Substituting the expression for $P(B)$:

$P(A) = 2 \times (2 P(D))$

$P(A) = 4 P(D)$

Now, substitute these expressions into the equation for the sum of probabilities:

$P(A) + P(B) + P(C) + P(D) = 1$

$4 P(D) + 2 P(D) + 2 P(D) + P(D) = 1$

Combine the terms:

$(4 + 2 + 2 + 1) P(D) = 1$

$9 P(D) = 1$

Solving for $P(D)$:

$P(D) = \frac{1}{9}$

Now we can find the probabilities for the other candidates:

$P(C) = 2 P(D) = 2 \times \frac{1}{9} = \frac{2}{9}$

$P(B) = P(C) = \frac{2}{9}$

$P(A) = 4 P(D) = 4 \times \frac{1}{9} = \frac{4}{9}$

Let's verify that the probabilities sum to 1:

$P(A) + P(B) + P(C) + P(D) = \frac{4}{9} + \frac{2}{9} + \frac{2}{9} + \frac{1}{9} = \frac{4+2+2+1}{9} = \frac{9}{9} = 1$. The probabilities are consistent.


(a) Probability that C will be selected:

We found $P(C) = \frac{2}{9}$.


(b) Probability that A will not be selected:

The event that A will not be selected is the complement of event A. The probability of the complement $A'$ is $P(A') = 1 - P(A)$.

$P(A') = 1 - \frac{4}{9}$

$P(A') = \frac{9}{9} - \frac{4}{9} = \frac{5}{9}$

Question 10. One of the four persons John, Rita, Aslam or Gurpreet will be promoted next month. Consequently the sample space consists of four elementary outcomes

S = {John promoted, Rita promoted, Aslam promoted, Gurpreet promoted}

You are told that the chances of John’s promotion is same as that of Gurpreet, Rita’s chances of promotion are twice as likely as Johns. Aslam’s chances are four times that of John.

(a) Determine P (John promoted)

P (Rita promoted)

P (Aslam promoted)

P (Gurpreet promoted)

(b) If A = {John promoted or Gurpreet promoted}, find P (A).

Answer:

Given:

The sample space $S = \{\text{John promoted, Rita promoted, Aslam promoted, Gurpreet promoted}\}$.

Let J, R, A, G represent the events that John, Rita, Aslam, or Gurpreet is promoted, respectively.

The given probability relationships are:

The chances of John’s promotion is same as that of Gurpreet: $P(J) = P(G)$.

Rita’s chances of promotion are twice as likely as Johns: $P(R) = 2 P(J)$.

Aslam’s chances are four times that of John: $P(A) = 4 P(J)$.


To Find:

(a) $P(\text{John promoted})$, $P(\text{Rita promoted})$, $P(\text{Aslam promoted})$, $P(\text{Gurpreet promoted})$.

(b) $P(A)$ where $A = \{\text{John promoted or Gurpreet promoted}\}$.


Solution:

The events J, R, A, and G are mutually exclusive and exhaustive (since exactly one person will be promoted). Therefore, the sum of their probabilities is 1.

$P(J) + P(R) + P(A) + P(G) = 1$

We can express the probabilities of R, A, and G in terms of $P(J)$ using the given relationships:

$P(R) = 2 P(J)$

$P(A) = 4 P(J)$

$P(G) = P(J)$

Substitute these expressions into the sum of probabilities equation:

$P(J) + 2 P(J) + 4 P(J) + P(J) = 1$

Combine the terms with $P(J)$:

$(1 + 2 + 4 + 1) P(J) = 1$

$8 P(J) = 1$

Solving for $P(J)$:

$P(J) = \frac{1}{8}$


(a) Determine individual probabilities:

$P(\text{John promoted}) = P(J) = \frac{1}{8}$

$P(\text{Rita promoted}) = P(R) = 2 P(J) = 2 \times \frac{1}{8} = \frac{2}{8} = \frac{1}{4}$

$P(\text{Aslam promoted}) = P(A) = 4 P(J) = 4 \times \frac{1}{8} = \frac{4}{8} = \frac{1}{2}$

$P(\text{Gurpreet promoted}) = P(G) = P(J) = \frac{1}{8}$


(b) Find P(A) where A = {John promoted or Gurpreet promoted}:

The event A is the union of events J and G, i.e., $A = J \cup G$.

Since J and G are mutually exclusive events (only one person can be promoted), the probability of their union is the sum of their individual probabilities.

$P(A) = P(J \cup G) = P(J) + P(G)$

Substitute the probabilities found in part (a):

$P(A) = \frac{1}{8} + \frac{1}{8}$

$P(A) = \frac{1+1}{8} = \frac{2}{8} = \frac{1}{4}$

Question 11. The accompanying Venn diagram shows three events, A, B, and C, and also the probabilities of the various intersections (for instance, P (A ∩ B) = .07). Determine

Page 298 Chapter 16 Class 11th NCERT Exemplar

(a) P (A)

(b) P (B ∩ $\overline{C}$)

(c) P (A ∪ B)

(d) P (A ∩ $\overline{B}$)

(e) P (B ∩ C)

(f) Probability of exactly one of the three occurs.

Answer:

Solution:

Based on the given Venn diagram, the probabilities of the disjoint regions are:

$P(A \cap B \cap C) = 0.01$

$P(A \cap B \cap C') = 0.06$

$P(A \cap B' \cap C) = 0.04$

$P(A' \cap B \cap C) = 0.05$

$P(A \cap B' \cap C') = 0.10$

$P(A' \cap B \cap C') = 0.13$

$P(A' \cap B' \cap C) = 0.17$

$P(A' \cap B' \cap C') = 0.44$

Let's verify the sum of probabilities: $0.01+0.06+0.04+0.05+0.10+0.13+0.17+0.44 = 1.00$.


(a) Determine P(A):

The probability of event A is the sum of the probabilities of all disjoint regions within A.

$P(A) = P(A \cap B \cap C) + P(A \cap B \cap C') + P(A \cap B' \cap C) + P(A \cap B' \cap C')$

$P(A) = 0.01 + 0.06 + 0.04 + 0.10$

$P(A) = 0.21$


(b) Determine P(B $\cap$ $\overline{C}$):

$\overline{C}$ is the complement of C, denoted as $C'$. $B \cap C'$ represents the region that is in B but not in C. This consists of the disjoint regions $A \cap B \cap C'$ and $A' \cap B \cap C'$.

$P(B \cap C') = P(A \cap B \cap C') + P(A' \cap B \cap C')$

$P(B \cap C') = 0.06 + 0.13$

$P(B \cap C') = 0.19$


(c) Determine P(A ∪ B):

The probability of the union of A and B is the sum of the probabilities of all disjoint regions within A or B or both.

$P(A \cup B) = P(A \cap B \cap C) + P(A \cap B \cap C') + P(A \cap B' \cap C) + P(A' \cap B \cap C) + P(A \cap B' \cap C') + P(A' \cap B \cap C')$

$P(A \cup B) = 0.01 + 0.06 + 0.04 + 0.05 + 0.10 + 0.13$

$P(A \cup B) = 0.39$

Alternatively, using the formula $P(A \cup B) = P(A) + P(B) - P(A \cap B)$:

First, calculate P(B):

$P(B) = P(A \cap B \cap C) + P(A \cap B \cap C') + P(A' \cap B \cap C) + P(A' \cap B \cap C')$

$P(B) = 0.01 + 0.06 + 0.05 + 0.13 = 0.25$

Next, calculate $P(A \cap B)$. This is the region common to A and B.

$P(A \cap B) = P(A \cap B \cap C) + P(A \cap B \cap C')$

$P(A \cap B) = 0.01 + 0.06 = 0.07$ (This matches the example given in the question).

Now, apply the formula:

$P(A \cup B) = P(A) + P(B) - P(A \cap B)$

$P(A \cup B) = 0.21 + 0.25 - 0.07$

$P(A \cup B) = 0.46 - 0.07 = 0.39$

Both methods yield the same result.


(d) Determine P(A $\cap$ $\overline{B}$):

$\overline{B}$ is the complement of B, denoted as $B'$. $A \cap B'$ represents the region that is in A but not in B. This consists of the disjoint regions $A \cap B' \cap C$ and $A \cap B' \cap C'$.

$P(A \cap B') = P(A \cap B' \cap C) + P(A \cap B' \cap C')$

$P(A \cap B') = 0.04 + 0.10$

$P(A \cap B') = 0.14$


(e) Determine P(B $\cap$ C):

The intersection $B \cap C$ is the region common to B and C. This consists of the disjoint regions $A \cap B \cap C$ and $A' \cap B \cap C$.

$P(B \cap C) = P(A \cap B \cap C) + P(A' \cap B \cap C)$

$P(B \cap C) = 0.01 + 0.05$

$P(B \cap C) = 0.06$


(f) Probability of exactly one of the three occurs:

This event corresponds to the union of the regions where only one of the events occurs. These disjoint regions are A only ($A \cap B' \cap C'$), B only ($A' \cap B \cap C'$), and C only ($A' \cap B' \cap C$).

Probability (exactly one) $= P(A \cap B' \cap C') + P(A' \cap B \cap C') + P(A' \cap B' \cap C)$

Probability (exactly one) $= 0.10 + 0.13 + 0.17$

Probability (exactly one) $= 0.40$

Question 12 to 17 (Long Answer Type Questions)

Question 12. One urn contains two black balls (labelled B1 and B2) and one white ball. A second urn contains one black ball and two white balls (labelled W1 and W2). Suppose the following experiment is performed. One of the two urns is chosen at random. Next a ball is randomly chosen from the urn. Then a second ball is chosen at random from the same urn without replacing the first ball.

(a) Write the sample space showing all possible outcomes

(b) What is the probability that two black balls are chosen?

(c) What is the probability that two balls of opposite colour are chosen?

Answer:

Given:

Urn 1 contains: 2 Black balls (B1, B2) and 1 White ball (W). Total 3 balls.

Urn 2 contains: 1 Black ball (B) and 2 White balls (W1, W2). Total 3 balls.

An experiment is performed: Choose an urn at random, then choose two balls without replacement from the chosen urn.


To Find:

(a) The sample space of all possible outcomes.

(b) The probability of choosing two black balls.

(c) The probability of choosing two balls of opposite colour.


Solution:

The experiment has two stages: selecting an urn and then drawing two balls from it without replacement. Since an urn is chosen at random, the probability of selecting Urn 1 is $\frac{1}{2}$, and the probability of selecting Urn 2 is $\frac{1}{2}$.


(a) Write the sample space showing all possible outcomes:

An outcome is represented by the sequence of choices: (Urn chosen, first ball drawn, second ball drawn).

Consider the outcomes when Urn 1 is chosen (containing B1, B2, W). There are 3 choices for the first ball and 2 choices for the second ball, giving $3 \times 2 = 6$ possible ordered draws:

(Urn 1, B1, B2)

(Urn 1, B1, W)

(Urn 1, B2, B1)

(Urn 1, B2, W)

(Urn 1, W, B1)

(Urn 1, W, B2)

Consider the outcomes when Urn 2 is chosen (containing B, W1, W2). There are 3 choices for the first ball and 2 choices for the second ball, giving $3 \times 2 = 6$ possible ordered draws:

(Urn 2, B, W1)

(Urn 2, B, W2)

(Urn 2, W1, B)

(Urn 2, W1, W2)

(Urn 2, W2, B)

(Urn 2, W2, W1)

The complete sample space S is the collection of all these 12 distinct outcomes:

$S = \{(Urn \; 1, B1, B2), (Urn \; 1, B1, W), (Urn \; 1, B2, B1), (Urn \; 1, B2, W), (Urn \; 1, W, B1), (Urn \; 1, W, B2), (Urn \; 2, B, W1), (Urn \; 2, B, W2), (Urn \; 2, W1, B), (Urn \; 2, W1, W2), (Urn \; 2, W2, B), (Urn \; 2, W2, W1)\}$


(b) What is the probability that two black balls are chosen?

Let $E_{BB}$ be the event that two black balls are chosen.

This event occurs if we choose Urn 1 and draw two black balls, OR choose Urn 2 and draw two black balls.

$P(E_{BB}) = P(\text{Urn 1 and 2 Black}) + P(\text{Urn 2 and 2 Black})$

Probability of choosing Urn 1 and drawing 2 black balls:

$P(\text{Urn 1 and 2 Black}) = P(\text{Choose Urn 1}) \times P(\text{Draw 2 Black | Urn 1})$

$P(\text{Choose Urn 1}) = \frac{1}{2}$

In Urn 1 (B1, B2, W), the probability of drawing a black ball first is $\frac{2}{3}$. Given the first is black, the probability of drawing a second black ball from the remaining 2 balls (1 black, 1 white) is $\frac{1}{2}$.

$P(\text{Draw 2 Black | Urn 1}) = P(\text{1st B}) \times P(\text{2nd B | 1st B}) = \frac{2}{3} \times \frac{1}{2} = \frac{2}{6} = \frac{1}{3}$

$P(\text{Urn 1 and 2 Black}) = \frac{1}{2} \times \frac{1}{3} = \frac{1}{6}$

Probability of choosing Urn 2 and drawing 2 black balls:

$P(\text{Urn 2 and 2 Black}) = P(\text{Choose Urn 2}) \times P(\text{Draw 2 Black | Urn 2})$

$P(\text{Choose Urn 2}) = \frac{1}{2}$

In Urn 2 (B, W1, W2), there is only 1 black ball. It is impossible to draw two black balls without replacement.

$P(\text{Draw 2 Black | Urn 2}) = 0$

$P(\text{Urn 2 and 2 Black}) = \frac{1}{2} \times 0 = 0$

Summing the probabilities:

$P(E_{BB}) = \frac{1}{6} + 0 = \frac{1}{6}$


(c) What is the probability that two balls of opposite colour are chosen?

Let $E_{Opp}$ be the event that two balls of opposite colour (one black and one white) are chosen.

This can happen by drawing a Black ball then a White ball (BW) or a White ball then a Black ball (WB).

$P(E_{Opp}) = P(\text{Urn 1 and Opposite}) + P(\text{Urn 2 and Opposite})$

Probability of choosing Urn 1 and drawing opposite colours:

$P(\text{Urn 1 and Opposite}) = P(\text{Choose Urn 1}) \times P(\text{Opposite | Urn 1})$

$P(\text{Choose Urn 1}) = \frac{1}{2}$

In Urn 1 (B1, B2, W), $P(\text{Opposite | Urn 1}) = P(\text{BW | Urn 1}) + P(\text{WB | Urn 1})$

$P(\text{BW | Urn 1}) = P(\text{1st B}) \times P(\text{2nd W | 1st B}) = \frac{2}{3} \times \frac{1}{2} = \frac{2}{6} = \frac{1}{3}$

$P(\text{WB | Urn 1}) = P(\text{1st W}) \times P(\text{2nd B | 1st W}) = \frac{1}{3} \times \frac{2}{2} = \frac{2}{6} = \frac{1}{3}$

$P(\text{Opposite | Urn 1}) = \frac{1}{3} + \frac{1}{3} = \frac{2}{3}$

$P(\text{Urn 1 and Opposite}) = \frac{1}{2} \times \frac{2}{3} = \frac{2}{6} = \frac{1}{3}$

Probability of choosing Urn 2 and drawing opposite colours:

$P(\text{Urn 2 and Opposite}) = P(\text{Choose Urn 2}) \times P(\text{Opposite | Urn 2})$

$P(\text{Choose Urn 2}) = \frac{1}{2}$

In Urn 2 (B, W1, W2), $P(\text{Opposite | Urn 2}) = P(\text{BW | Urn 2}) + P(\text{WB | Urn 2})$

$P(\text{BW | Urn 2}) = P(\text{1st B}) \times P(\text{2nd W | 1st B}) = \frac{1}{3} \times \frac{2}{2} = \frac{2}{6} = \frac{1}{3}$

$P(\text{WB | Urn 2}) = P(\text{1st W}) \times P(\text{2nd B | 1st W}) = \frac{2}{3} \times \frac{1}{2} = \frac{2}{6} = \frac{1}{3}$

$P(\text{Opposite | Urn 2}) = \frac{1}{3} + \frac{1}{3} = \frac{2}{3}$

$P(\text{Urn 2 and Opposite}) = \frac{1}{2} \times \frac{2}{3} = \frac{2}{6} = \frac{1}{3}$

Summing the probabilities:

$P(E_{Opp}) = \frac{1}{3} + \frac{1}{3} = \frac{2}{3}$


Final Answers:

(a) The sample space S is:

\{(Urn 1, B1, B2), (Urn 1, B1, W), (Urn 1, B2, B1), (Urn 1, B2, W), (Urn 1, W, B1), (Urn 1, W, B2), (Urn 2, B, W1), (Urn 2, B, W2), (Urn 2, W1, B), (Urn 2, W1, W2), (Urn 2, W2, B), (Urn 2, W2, W1)\}

(b) The probability that two black balls are chosen is $\frac{1}{6}$.

(c) The probability that two balls of opposite colour are chosen is $\frac{2}{3}$.

Question 13. A bag contains 8 red and 5 white balls. Three balls are drawn at random. Find the Probability that

(a) All the three balls are white

(b) All the three balls are red

(c) One ball is red and two balls are white

Answer:

Given:

Number of red balls = 8

Number of white balls = 5

Total number of balls = 8 + 5 = 13

Three balls are drawn at random from the bag.


To Find:

(a) The probability that all three balls drawn are white.

(b) The probability that all three balls drawn are red.

(c) The probability that one ball is red and two balls are white.


Solution:

When three balls are drawn at random from the bag, the total number of possible outcomes is the number of ways to choose 3 balls from 13, without regard to order. This can be calculated using combinations:

Total number of ways to draw 3 balls from 13 = $\binom{13}{3}$

$\binom{13}{3} = \frac{13!}{3!(13-3)!} = \frac{13!}{3!10!}$

$\binom{13}{3} = \frac{13 \times 12 \times 11}{3 \times 2 \times 1} = 13 \times 2 \times 11 = 286$

Total number of possible outcomes in the sample space = 286.


(a) Probability that all three balls are white:

This event occurs if we choose 3 white balls from the 5 available white balls.

Number of ways to choose 3 white balls from 5 = $\binom{5}{3}$

$\binom{5}{3} = \frac{5!}{3!(5-3)!} = \frac{5!}{3!2!} = \frac{5 \times 4}{2 \times 1} = 10$

Number of favorable outcomes for event (a) = 10.

The probability of this event is the number of favorable outcomes divided by the total number of possible outcomes.

P(All three balls are white) = $\frac{\text{Number of ways to choose 3 white}}{\text{Total number of ways to choose 3}}$

P(All three balls are white) = $\frac{10}{286}$

Simplifying the fraction by dividing both numerator and denominator by 2:

P(All three balls are white) = $\frac{\cancel{10}^{5}}{\cancel{286}_{143}} = \frac{5}{143}$


(b) Probability that all three balls are red:

This event occurs if we choose 3 red balls from the 8 available red balls.

Number of ways to choose 3 red balls from 8 = $\binom{8}{3}$

$\binom{8}{3} = \frac{8!}{3!(8-3)!} = \frac{8!}{3!5!} = \frac{8 \times 7 \times 6}{3 \times 2 \times 1} = 56$

Number of favorable outcomes for event (b) = 56.

P(All three balls are red) = $\frac{\text{Number of ways to choose 3 red}}{\text{Total number of ways to choose 3}}$

P(All three balls are red) = $\frac{56}{286}$

Simplifying the fraction by dividing both numerator and denominator by 2:

P(All three balls are red) = $\frac{\cancel{56}^{28}}{\cancel{286}_{143}} = \frac{28}{143}$


(c) Probability that one ball is red and two balls are white:

This event occurs if we choose 1 red ball from the 8 red balls AND 2 white balls from the 5 white balls.

Number of ways to choose 1 red ball from 8 = $\binom{8}{1} = 8$

Number of ways to choose 2 white balls from 5 = $\binom{5}{2} = \frac{5!}{2!(5-2)!} = \frac{5!}{2!3!} = \frac{5 \times 4}{2 \times 1} = 10$

The number of ways to choose 1 red and 2 white balls is the product of the number of ways to make each selection:

Number of favorable outcomes for event (c) = $\binom{8}{1} \times \binom{5}{2} = 8 \times 10 = 80$

P(One red and two white) = $\frac{\text{Number of ways to choose 1 red and 2 white}}{\text{Total number of ways to choose 3}}$

P(One red and two white) = $\frac{80}{286}$

Simplifying the fraction by dividing both numerator and denominator by 2:

P(One red and two white) = $\frac{\cancel{80}^{40}}{\cancel{286}_{143}} = \frac{40}{143}$


Final Answers:

(a) P(All three balls are white) = $\frac{5}{143}$.

(b) P(All three balls are red) = $\frac{28}{143}$.

(c) P(One ball is red and two balls are white) = $\frac{40}{143}$.

Question 14. If the letters of the word ASSASSINATION are arranged at random. Find the Probability that

(a) Four S’s come consecutively in the word

(b) Two I’s and two N’s come together

(c) All A’s are not coming together

(d) No two A’s are coming together.

Answer:

Given:

The word is ASSASSINATION.

The letters are arranged at random in a row.

The letters in the word are A(3), S(4), I(2), N(2), T(1), O(1).

Total number of letters = $3 + 4 + 2 + 2 + 1 + 1 = 13$.


To Find:

(a) Probability that the four S’s come consecutively.

(b) Probability that the two I’s and two N’s come together.

(c) Probability that all A’s are not coming together.

(d) Probability that no two A’s are coming together.


Solution:

First, calculate the total number of distinct arrangements of the letters in the word ASSASSINATION.

The word has 13 letters with repetitions: 3 A's, 4 S's, 2 I's, 2 N's, 1 T, 1 O.

The total number of permutations is given by the formula $\frac{n!}{n_1! n_2! \dots n_k!}$, where $n$ is the total number of items and $n_i$ is the count of identical items of type $i$.

Total number of arrangements $= \frac{13!}{3! 4! 2! 2! 1! 1!} = \frac{13!}{3! 4! 2! 2!}$

Let $N$ denote the total number of arrangements.

$N = \frac{13!}{3! 4! 2! 2!} = \frac{6,227,020,800}{(6)(24)(2)(2)} = \frac{6,227,020,800}{576} = 10,810,800$


(a) Probability that the four S’s come consecutively:

Treat the four S's as a single block (SSSS). Now we are arranging 10 entities: the block (SSSS) and the remaining 9 letters (A, A, A, I, I, N, N, T, O).

The number of arrangements of these 10 entities, considering the repetitions of A (3 times), I (2 times), and N (2 times), is:

Number of favorable arrangements $= \frac{10!}{3! 2! 2! 1! 1!} = \frac{10!}{3! 2! 2!}$

Let $N_a$ be the number of favorable arrangements.

$N_a = \frac{3,628,800}{(6)(2)(2)} = \frac{3,628,800}{24} = 151,200$

The probability is the ratio of favorable arrangements to the total number of arrangements.

$P(\text{SSSS together}) = \frac{N_a}{N} = \frac{151200}{10810800}$

Simplifying the fraction:

$P(\text{SSSS together}) = \frac{\cancel{151200}^{1512}}{\cancel{10810800}_{108108}} = \frac{\cancel{1512}^{126}}{\cancel{108108}_{9009}} = \frac{\cancel{126}^{14}}{\cancel{9009}_{1001}} = \frac{\cancel{14}^{2}}{\cancel{1001}_{143}} = \frac{2}{143}$


(b) Probability that the two I’s and two N’s come together:

Interpret this as the two I's forming a unit (II) and the two N's forming a unit (NN). Now we are arranging 11 entities: the blocks (II), (NN), and the remaining 9 letters (A, A, A, S, S, S, S, T, O).

The number of arrangements of these 11 entities, considering the repetitions of A (3 times) and S (4 times), is:

Number of favorable arrangements $= \frac{11!}{3! 4! 1! 1! 1!} = \frac{11!}{3! 4!}$

Let $N_b$ be the number of favorable arrangements.

$N_b = \frac{39,916,800}{(6)(24)} = \frac{39,916,800}{144} = 277,200$

The probability is the ratio of favorable arrangements to the total number of arrangements.

$P(\text{II and NN together}) = \frac{N_b}{N} = \frac{277200}{10810800}$

Simplifying the fraction:

$P(\text{II and NN together}) = \frac{\cancel{277200}^{2772}}{\cancel{10810800}_{108108}} = \frac{\cancel{2772}^{231}}{\cancel{108108}_{9009}} = \frac{\cancel{231}^{1}}{\cancel{9009}_{39}} = \frac{1}{39}$


(c) Probability that all A’s are not coming together:

This is the complement of the event that all A's come together. Let $E'$ be the event that all A's are coming together.

$P(\text{All A's not together}) = 1 - P(E')$

To find $P(E')$, treat the three A's as a single block (AAA). Now we are arranging 11 entities: the block (AAA) and the remaining 10 letters (S, S, S, S, I, I, N, N, T, O).

The number of arrangements of these 11 entities, considering the repetitions of S (4 times), I (2 times), and N (2 times), is:

Number of arrangements with AAA together $= \frac{11!}{4! 2! 2! 1! 1!} = \frac{11!}{4! 2! 2!}$

Let $N_{c'}$ be the number of favorable arrangements for $E'$.

$N_{c'} = \frac{39,916,800}{(24)(2)(2)} = \frac{39,916,800}{96} = 415,800$

The probability of event $E'$ is:

$P(E') = \frac{N_{c'}}{N} = \frac{415800}{10810800}$

Simplifying the fraction:

$P(E') = \frac{\cancel{415800}^{4158}}{\cancel{10810800}_{108108}} = \frac{\cancel{4158}^{231}}{\cancel{108108}_{6006}} = \frac{\cancel{231}^{77}}{\cancel{6006}_{2002}} = \frac{\cancel{77}^{11}}{\cancel{2002}_{286}} = \frac{\cancel{11}^{1}}{\cancel{286}_{26}} = \frac{1}{26}$

Now calculate the probability that all A's are not coming together:

$P(\text{All A's not together}) = 1 - P(E') = 1 - \frac{1}{26} = \frac{26 - 1}{26} = \frac{25}{26}$


(d) Probability that no two A’s are coming together:

To ensure no two A's are adjacent, first arrange the other 10 letters (S, S, S, S, I, I, N, N, T, O).

Number of arrangements of the 10 non-A letters $= \frac{10!}{4! 2! 2! 1! 1!} = \frac{10!}{4! 2! 2!}$

$= \frac{3,628,800}{(24)(2)(2)} = \frac{3,628,800}{96} = 37,800$

When these 10 letters are arranged, they create 11 spaces where the three A's can be placed so that no two A's are adjacent.

_ L _ L _ L _ L _ L _ L _ L _ L _ L _ L _

Here, 'L' represents a non-A letter, and '_' represents a potential space for an A.

We need to choose 3 of these 11 spaces for the three identical A's. The number of ways to choose 3 spaces from 11 is given by the combination formula $\binom{n}{k} = \frac{n!}{k!(n-k)!}$.

Number of ways to choose spaces for A's $= \binom{11}{3} = \frac{11!}{3!(11-3)!} = \frac{11!}{3!8!} = \frac{11 \times 10 \times 9}{3 \times 2 \times 1} = 11 \times 5 \times 3 = 165$

The number of favorable arrangements (where no two A's are together) is the product of the number of ways to arrange the non-A letters and the number of ways to place the A's in the spaces.

Number of favorable arrangements $= (\text{Arrangements of non-A letters}) \times (\text{Ways to place A's})$

Number of favorable arrangements $= 37800 \times 165 = 6,237,000$

Let $N_d$ be the number of favorable arrangements.

$P(\text{No two A's together}) = \frac{N_d}{N} = \frac{6237000}{10810800}$

Simplifying the fraction:

$P(\text{No two A's together}) = \frac{\cancel{6237000}^{62370}}{\cancel{10810800}_{108108}} = \frac{\cancel{62370}^{3465}}{\cancel{108108}_{6006}} = \frac{\cancel{3465}^{1155}}{\cancel{6006}_{2002}} = \frac{\cancel{1155}^{165}}{\cancel{2002}_{286}} = \frac{\cancel{165}^{15}}{\cancel{286}_{26}} = \frac{15}{26}$

Question 15. A card is drawn from a deck of 52 cards. Find the probability of getting a king or a heart or a red card.

Answer:

Given:

A standard deck of 52 cards.

One card is drawn at random.


To Find:

The probability of getting a king or a heart or a red card.


Solution:

Let S be the sample space of drawing a card from a deck of 52 cards.

Total number of outcomes in S is $|S| = 52$.

Let K be the event of getting a King.

Let H be the event of getting a Heart.

Let R be the event of getting a Red card.

We want to find the probability of the union of these three events, $P(K \cup H \cup R)$.

The number of outcomes in each event and their intersections are:

Number of Kings, $|K| = 4$

Number of Hearts, $|H| = 13$

Number of Red cards, $|R| = 26$ (13 Hearts + 13 Diamonds)

Number of Kings that are Hearts, $|K \cap H| = 1$ (King of Hearts)

Number of Kings that are Red, $|K \cap R| = 2$ (King of Hearts, King of Diamonds)

Number of Hearts that are Red, $|H \cap R| = 13$ (All Hearts are Red)

Number of cards that are King AND Heart AND Red, $|K \cap H \cap R| = 1$ (King of Hearts)

Using the Principle of Inclusion-Exclusion for three events:

$P(K \cup H \cup R) = P(K) + P(H) + P(R) - P(K \cap H) - P(K \cap R) - P(H \cap R) + P(K \cap H \cap R)$

The probabilities are:

$P(K) = \frac{|K|}{|S|} = \frac{4}{52}$

$P(H) = \frac{|H|}{|S|} = \frac{13}{52}$

$P(R) = \frac{|R|}{|S|} = \frac{26}{52}$

$P(K \cap H) = \frac{|K \cap H|}{|S|} = \frac{1}{52}$

$P(K \cap R) = \frac{|K \cap R|}{|S|} = \frac{2}{52}$

$P(H \cap R) = \frac{|H \cap R|}{|S|} = \frac{13}{52}$

$P(K \cap H \cap R) = \frac{|K \cap H \cap R|}{|S|} = \frac{1}{52}$

Substitute these values into the formula:

$P(K \cup H \cup R) = \frac{4}{52} + \frac{13}{52} + \frac{26}{52} - \frac{1}{52} - \frac{2}{52} - \frac{13}{52} + \frac{1}{52}$

$P(K \cup H \cup R) = \frac{4 + 13 + 26 - 1 - 2 - 13 + 1}{52}$

$P(K \cup H \cup R) = \frac{44 - 16}{52} = \frac{28}{52}$

Simplify the fraction:

$P(K \cup H \cup R) = \frac{\cancel{28}^{7}}{\cancel{52}_{13}} = \frac{7}{13}$


Alternate Solution:

Notice that all Heart cards are also Red cards. This means the set of Hearts (H) is a subset of the set of Red cards (R), i.e., $H \subseteq R$.

When finding the union of events K, H, and R, since H is included in R, the event "Heart or Red" ($H \cup R$) is simply the event "Red" (R).

Therefore, the event "King or Heart or Red" ($K \cup H \cup R$) is equivalent to the event "King or Red" ($K \cup R$).

We need to find $P(K \cup R)$. Using the formula for the union of two events:

$P(K \cup R) = P(K) + P(R) - P(K \cap R)$

We have:

$P(K) = \frac{4}{52}$

$P(R) = \frac{26}{52}$

$P(K \cap R)$ is the probability of getting a King and a Red card. There are 2 Red Kings (King of Hearts, King of Diamonds).

$P(K \cap R) = \frac{2}{52}$

Substitute these values into the formula:

$P(K \cup R) = \frac{4}{52} + \frac{26}{52} - \frac{2}{52}$

$P(K \cup R) = \frac{4 + 26 - 2}{52} = \frac{30 - 2}{52} = \frac{28}{52}$

Simplify the fraction:

$P(K \cup R) = \frac{\cancel{28}^{7}}{\cancel{52}_{13}} = \frac{7}{13}$


The probability of getting a king or a heart or a red card is $\frac{7}{13}$.

Question 16. A sample space consists of 9 elementary outcomes e1, e2, ..., e9 whose probabilitiesare

P(e1) = P(e2) = .08

P(e3) = P(e4) = P(e5) = .1

P(e6) = P(e7) = .2

P(e8) = P(e9) = .07

Suppose A = {e1, e5, e8}, B = {e2, e5, e8, e9}

(a) Calculate P (A), P (B), and P (A ∩ B)

(b) Using the addition law of probability, calculate P (A∪ B)

(c) List the composition of the event A ∪ B, and calculate P (A ∪ B) by adding the probabilities of the elementary outcomes.

(d) Calculate P ($\overline{B}$) from P (B), also calculate P ($\overline{B}$) directly from the elementary outcomes of $\overline{B}$

Answer:

Given:

Sample space $S = \{e_1, e_2, e_3, e_4, e_5, e_6, e_7, e_8, e_9\}$.

Probabilities of elementary outcomes:

$P(e_1) = 0.08$, $P(e_2) = 0.08$

$P(e_3) = 0.10$, $P(e_4) = 0.10$, $P(e_5) = 0.10$

$P(e_6) = 0.20$, $P(e_7) = 0.20$

$P(e_8) = 0.07$, $P(e_9) = 0.07$

Events $A = \{e_1, e_5, e_8\}$ and $B = \{e_2, e_5, e_8, e_9\}$.


To Find:

(a) $P(A)$, $P(B)$, and $P(A \cap B)$.

(b) $P(A \cup B)$ using the addition law.

(c) Composition of $A \cup B$ and $P(A \cup B)$ by adding probabilities.

(d) $P(\overline{B})$ from $P(B)$ and directly from elementary outcomes of $\overline{B}$.


Solution:

The probability of an event is the sum of the probabilities of the elementary outcomes comprising the event.


(a) Calculate P(A), P(B), and P(A $\cap$ B):

$A = \{e_1, e_5, e_8\}$

$P(A) = P(e_1) + P(e_5) + P(e_8)$

$P(A) = 0.08 + 0.10 + 0.07 = 0.25$

$B = \{e_2, e_5, e_8, e_9\}$

$P(B) = P(e_2) + P(e_5) + P(e_8) + P(e_9)$

$P(B) = 0.08 + 0.10 + 0.07 + 0.07 = 0.32$

The intersection of A and B, $A \cap B$, consists of the elementary outcomes common to both A and B.

$A \cap B = \{e_1, e_5, e_8\} \cap \{e_2, e_5, e_8, e_9\} = \{e_5, e_8\}$

$P(A \cap B) = P(e_5) + P(e_8)$

$P(A \cap B) = 0.10 + 0.07 = 0.17$


(b) Using the addition law of probability, calculate P(A $\cup$ B):

The addition law of probability states that for any two events A and B:

$P(A \cup B) = P(A) + P(B) - P(A \cap B)$

Using the values calculated in part (a):

$P(A \cup B) = 0.25 + 0.32 - 0.17$

$P(A \cup B) = 0.57 - 0.17 = 0.40$


(c) List the composition of the event A $\cup$ B, and calculate P(A $\cup$ B) by adding the probabilities of the elementary outcomes:

The union of A and B, $A \cup B$, consists of the elementary outcomes that are in A or in B or in both.

$A \cup B = \{e_1, e_5, e_8\} \cup \{e_2, e_5, e_8, e_9\} = \{e_1, e_2, e_5, e_8, e_9\}$

Now, calculate $P(A \cup B)$ by summing the probabilities of these elementary outcomes:

$P(A \cup B) = P(e_1) + P(e_2) + P(e_5) + P(e_8) + P(e_9)$

$P(A \cup B) = 0.08 + 0.08 + 0.10 + 0.07 + 0.07$

$P(A \cup B) = 0.16 + 0.10 + 0.14 = 0.40$

This result matches the result obtained using the addition law in part (b).


(d) Calculate P($\overline{B}$) from P(B), also calculate P($\overline{B}$) directly from the elementary outcomes of $\overline{B}$:

Calculate $P(\overline{B})$ from $P(B)$ using the complement rule: $P(\overline{B}) = 1 - P(B)$.

From part (a), $P(B) = 0.32$.

$P(\overline{B}) = 1 - 0.32 = 0.68$

Now, calculate $P(\overline{B})$ directly from the elementary outcomes of $\overline{B}$. The complement of B, $\overline{B}$, consists of all elementary outcomes in the sample space S that are not in B.

$S = \{e_1, e_2, e_3, e_4, e_5, e_6, e_7, e_8, e_9\}$

$B = \{e_2, e_5, e_8, e_9\}$

$\overline{B} = \{e_1, e_3, e_4, e_6, e_7\}$

Calculate $P(\overline{B})$ by summing the probabilities of these elementary outcomes:

$P(\overline{B}) = P(e_1) + P(e_3) + P(e_4) + P(e_6) + P(e_7)$

$P(\overline{B}) = 0.08 + 0.10 + 0.10 + 0.20 + 0.20$

$P(\overline{B}) = 0.08 + 0.20 + 0.40 = 0.68$

Both methods give the same result for $P(\overline{B})$.

Question 17. Determine the probability p, for each of the following events.

(a) An odd number appears in a single toss of a fair die.

(b) At least one head appears in two tosses of a fair coin.

(c) A king, 9 of hearts, or 3 of spades appears in drawing a single card from a well shuffled ordinary deck of 52 cards.

(d) The sum of 6 appears in a single toss of a pair of fair dice.

Answer:

Solution (a):


Given:

A single toss of a fair die.

Event: An odd number appears.


To Find:

The probability of getting an odd number.


Solution:

The sample space for a single toss of a fair die is $S = \{1, 2, 3, 4, 5, 6\}$.

The total number of possible outcomes is $|S| = 6$.

Let E be the event that an odd number appears. The odd numbers in the sample space are 1, 3, and 5.

The favorable outcomes for event E are $\{1, 3, 5\}$.

The number of favorable outcomes is $|E| = 3$.

The probability of event E is given by:

$P(E) = \frac{\text{Number of favorable outcomes}}{\text{Total number of possible outcomes}}$

$p = \frac{|E|}{|S|} = \frac{3}{6}$

Simplify the fraction:

$p = \frac{\cancel{3}^{1}}{\cancel{6}_{2}} = \frac{1}{2}$


Solution (b):


Given:

Two tosses of a fair coin.

Event: At least one head appears.


To Find:

The probability of getting at least one head.


Solution:

The sample space for two tosses of a fair coin is $S = \{HH, HT, TH, TT\}$.

The total number of possible outcomes is $|S| = 4$.

Let E be the event that at least one head appears. The outcomes in E are those that contain at least one H.

The favorable outcomes for event E are $\{HH, HT, TH\}$.

The number of favorable outcomes is $|E| = 3$.

The probability of event E is given by:

$p = \frac{\text{Number of favorable outcomes}}{\text{Total number of possible outcomes}}$

$p = \frac{|E|}{|S|} = \frac{3}{4}$


Solution (c):


Given:

Drawing a single card from a well-shuffled ordinary deck of 52 cards.

Event: Getting a king or a 9 of hearts or a 3 of spades.


To Find:

The probability of getting a king or a 9 of hearts or a 3 of spades.


Solution:

The total number of possible outcomes when drawing a single card from a deck of 52 is 52.

Total number of possible outcomes = 52.

Let K be the event of getting a King.

Let H9 be the event of getting the 9 of hearts.

Let S3 be the event of getting the 3 of spades.

We are looking for the probability of the event $K \cup H9 \cup S3$.

Number of Kings in a deck = 4.

Number of 9 of hearts = 1.

Number of 3 of spades = 1.

These three events (getting a King, getting the 9 of hearts, getting the 3 of spades) are mutually exclusive, as no card can be both a King, a 9 of hearts, and a 3 of spades simultaneously.

The number of favorable outcomes is the sum of the number of outcomes in each event:

Number of favorable outcomes $= |K| + |H9| + |S3| = 4 + 1 + 1 = 6$.

The probability is given by:

$p = \frac{\text{Number of favorable outcomes}}{\text{Total number of possible outcomes}}$

$p = \frac{6}{52}$

Simplify the fraction:

$p = \frac{\cancel{6}^{3}}{\cancel{52}_{26}} = \frac{3}{26}$


Solution (d):


Given:

A single toss of a pair of fair dice.

Event: The sum of the numbers is 6.


To Find:

The probability that the sum of the numbers is 6.


Solution:

When a pair of fair dice is tossed, the total number of possible outcomes is $6 \times 6 = 36$.

Total number of possible outcomes = 36.

Let E be the event that the sum of the numbers is 6. The pairs of numbers that sum to 6 are:

$(1, 5)$ (Die 1 shows 1, Die 2 shows 5)

$(2, 4)$

$(3, 3)$

$(4, 2)$

$(5, 1)$

The favorable outcomes for event E are $\{(1, 5), (2, 4), (3, 3), (4, 2), (5, 1)\}$.

The number of favorable outcomes is $|E| = 5$.

The probability of event E is given by:

$p = \frac{\text{Number of favorable outcomes}}{\text{Total number of possible outcomes}}$

$p = \frac{|E|}{|S|} = \frac{5}{36}$

Question 18 to 29 (Multiple Choice Questions)

Choose the correct answer out of four given options in each of the Exercises 18 to 29 (M.C.Q.).

Question 18. In a non-leap year, the probability of having 53 tuesdays or 53 wednesdays is

(A) $\frac{1}{7}$

(B) $\frac{2}{7}$

(C) $\frac{3}{7}$

(D) none of these

Answer:

Given:

A non-leap year.

The event is having 53 Tuesdays or 53 Wednesdays.


To Find:

The probability of the given event.


Solution:

A non-leap year has 365 days.

We know that $365 = 52 \times 7 + 1$.

This means a non-leap year has 52 full weeks and 1 extra day.

The 52 full weeks contain exactly 52 Tuesdays and 52 Wednesdays.

The occurrence of a 53rd Tuesday or a 53rd Wednesday depends entirely on the nature of the single extra day.

The possible outcomes for the extra day are the 7 days of the week: {Sunday, Monday, Tuesday, Wednesday, Thursday, Friday, Saturday}.

Total number of possible outcomes for the extra day = 7.

Let A be the event that the extra day is a Tuesday. If the extra day is Tuesday, the year will have 53 Tuesdays.

The number of favorable outcomes for event A = 1 (Tuesday).

$P(\text{53 Tuesdays}) = P(\text{extra day is Tuesday}) = \frac{1}{7}$.

Let B be the event that the extra day is a Wednesday. If the extra day is Wednesday, the year will have 53 Wednesdays.

The number of favorable outcomes for event B = 1 (Wednesday).

$P(\text{53 Wednesdays}) = P(\text{extra day is Wednesday}) = \frac{1}{7}$.

We are looking for the probability of having 53 Tuesdays OR 53 Wednesdays. This is the probability of the union of events A and B, i.e., $P(A \cup B)$.

Events A and B are mutually exclusive because the extra day cannot be both Tuesday and Wednesday simultaneously.

For mutually exclusive events, $P(A \cup B) = P(A) + P(B)$.

$P(\text{53 Tuesdays or 53 Wednesdays}) = P(\text{53 Tuesdays}) + P(\text{53 Wednesdays})$

$P(A \cup B) = \frac{1}{7} + \frac{1}{7} = \frac{2}{7}$.


The probability of having 53 Tuesdays or 53 Wednesdays in a non-leap year is $\frac{2}{7}$.

This corresponds to option (B).

Question 19. Three numbers are chosen from 1 to 20. Find the probability that they are not consecutive

(A) $\frac{186}{190}$

(B) $\frac{187}{190}$

(C) $\frac{188}{190}$

(D) $\frac{18}{^{20}C_3}$

Answer:

Given:

Three numbers are chosen from the integers 1 to 20.


To Find:

The probability that the three chosen numbers are not consecutive.


Solution:

Let S be the sample space of choosing 3 numbers from the 20 integers {1, 2, ..., 20}. The total number of ways to choose 3 numbers from 20 is given by the combination formula $\binom{n}{k} = \frac{n!}{k!(n-k)!}$.

Total number of outcomes = $\binom{20}{3}$

$\binom{20}{3} = \frac{20 \times 19 \times 18}{3 \times 2 \times 1} = \frac{6840}{6} = 1140$

Total number of possible outcomes = 1140.


Let A be the event that the three chosen numbers are consecutive.

Consecutive sets of 3 numbers from 1 to 20 are of the form $(n, n+1, n+2)$.

The possible values for the first number $n$ are from 1 up to the point where $n+2 \leq 20$.

$n+2 \leq 20 \implies n \leq 18$.

So, the possible consecutive sets are:

(1, 2, 3)

(2, 3, 4)

...

(18, 19, 20)

The number of such consecutive sets is equal to the number of possible values for $n$, which is 18.

Number of favorable outcomes for event A = 18.

The probability of event A (getting 3 consecutive numbers) is:

$P(A) = \frac{\text{Number of consecutive sets}}{\text{Total number of sets}} = \frac{18}{1140}$


We are asked to find the probability that the three chosen numbers are not consecutive. This is the complement of event A, denoted as $A'$.

$P(A') = P(\text{not consecutive}) = 1 - P(A)$

$P(A') = 1 - \frac{18}{1140}$

$P(A') = \frac{1140 - 18}{1140} = \frac{1122}{1140}$

Simplify the fraction by dividing the numerator and denominator by their greatest common divisor. Both are divisible by 6.

$1122 \div 6 = 187$

$1140 \div 6 = 190$

$P(A') = \frac{\cancel{1122}^{187}}{\cancel{1140}_{190}} = \frac{187}{190}$


The probability that the three chosen numbers are not consecutive is $\frac{187}{190}$.

Comparing this result with the given options, we find that it matches option (B).

Question 20. While shuffling a pack of 52 playing cards, 2 are accidentally dropped. Find the probability that the missing cards to be of different colours

(A) $\frac{29}{52}$

(B) $\frac{1}{2}$

(C) $\frac{26}{51}$

(D) $\frac{27}{51}$

Answer:

Given:

A pack of 52 playing cards.

2 cards are accidentally dropped (chosen) from the pack.


To Find:

The probability that the two missing cards are of different colours.


Solution:

A standard deck of 52 cards consists of 26 red cards and 26 black cards.

When 2 cards are dropped, we are essentially choosing 2 cards from the deck without replacement and without considering the order.

The total number of ways to choose 2 cards from 52 is given by the combination formula $\binom{n}{k} = \frac{n!}{k!(n-k)!}$.

Total number of ways to choose 2 cards from 52 = $\binom{52}{2}$

$\binom{52}{2} = \frac{52 \times 51}{2 \times 1} = 26 \times 51 = 1326$

Total number of possible outcomes = 1326.

We want to find the number of ways to choose 2 cards such that they are of different colours. This means one card is red and the other card is black.

Number of ways to choose 1 red card from 26 red cards = $\binom{26}{1} = 26$.

Number of ways to choose 1 black card from 26 black cards = $\binom{26}{1} = 26$.

The number of ways to choose 1 red card and 1 black card is the product of the number of ways to make each choice.

Number of favorable outcomes = $\binom{26}{1} \times \binom{26}{1} = 26 \times 26 = 676$

The probability that the two missing cards are of different colours is the ratio of the number of favorable outcomes to the total number of possible outcomes.

Probability (different colours) = $\frac{\text{Number of ways to choose 1 red and 1 black}}{\text{Total number of ways to choose 2 cards}}$

Probability = $\frac{676}{1326}$

Simplify the fraction by dividing both numerator and denominator by their greatest common divisor. Both are divisible by 26.

$676 \div 26 = 26$

$1326 \div 26 = 51$

Probability = $\frac{\cancel{676}^{26}}{\cancel{1326}_{51}} = \frac{26}{51}$


The probability that the missing cards are of different colours is $\frac{26}{51}$.

This corresponds to option (C).

Question 21. Seven persons are to be seated in a row. The probability that two particular persons sit next to each other is

(A) $\frac{1}{3}$

(B) $\frac{1}{6}$

(C) $\frac{2}{7}$

(D) $\frac{1}{2}$

Answer:

Given:

Seven persons are to be seated in a row.


To Find:

The probability that two particular persons sit next to each other.


Solution:

Let the seven persons be $P_1, P_2, P_3, P_4, P_5, P_6, P_7$. Let the two particular persons be $P_1$ and $P_2$.

Total Number of Arrangements:

The total number of ways to arrange 7 distinct persons in a row is $7!$.

Total number of arrangements $= 7! = 7 \times 6 \times 5 \times 4 \times 3 \times 2 \times 1 = 5040$


Favorable Arrangements (Two particular persons sit together):

Consider the two particular persons ($P_1$ and $P_2$) as a single unit. Now we are arranging this unit and the remaining 5 persons, making a total of 6 entities to arrange.

The number of ways to arrange these 6 entities is $6!$.

$6! = 6 \times 5 \times 4 \times 3 \times 2 \times 1 = 720$

Within the unit of the two particular persons, they can be arranged in $2!$ ways ($P_1P_2$ or $P_2P_1$).

$2! = 2 \times 1 = 2$

The total number of arrangements where the two particular persons sit next to each other is the product of the number of ways to arrange the 6 entities and the number of ways to arrange the persons within the unit.

Number of favorable arrangements $= 6! \times 2! = 720 \times 2 = 1440$


Probability:

The probability that the two particular persons sit next to each other is the ratio of the number of favorable arrangements to the total number of arrangements.

Probability $= \frac{\text{Number of favorable arrangements}}{\text{Total number of arrangements}}$

Probability $= \frac{6! \times 2!}{7!}$

We can write $7!$ as $7 \times 6!$.

Probability $= \frac{6! \times 2}{7 \times 6!}$

Cancel out $6!$ from the numerator and denominator.

Probability $= \frac{2}{7}$


The probability that two particular persons sit next to each other when seven persons are seated in a row is $\frac{2}{7}$.

This corresponds to option (C).

Question 22. Without repetition of the numbers, four digit numbers are formed with the numbers 0, 2, 3, 5. The probability of such a number divisible by 5 is

(A) $\frac{1}{5}$

(B) $\frac{4}{5}$

(C) $\frac{1}{30}$

(D) $\frac{5}{9}$

Answer:

Given:

The digits available are 0, 2, 3, 5.

Four-digit numbers are formed using these digits without repetition.


To Find:

The probability that a randomly formed four-digit number is divisible by 5.


Solution:

First, we need to find the total number of four-digit numbers that can be formed using the digits 0, 2, 3, 5 without repetition. A four-digit number cannot have 0 in the thousands place.

Number of choices for the thousands place = 3 (can be 2, 3, or 5).

Number of choices for the hundreds place = 3 (any of the remaining 3 digits).

Number of choices for the tens place = 2 (any of the remaining 2 digits).

Number of choices for the units place = 1 (the last remaining digit).

Total number of four-digit numbers = $3 \times 3 \times 2 \times 1 = 18$.

Total number of possible outcomes = 18.


Next, we need to find the number of these four-digit numbers that are divisible by 5. A number is divisible by 5 if its units digit is 0 or 5.

Case 1: The units digit is 0.

Units place: 1 choice (0).

Thousands place: 3 choices (2, 3, 5, as 0 is used).

Hundreds place: 2 choices (remaining 2 digits).

Tens place: 1 choice (remaining 1 digit).

Number of four-digit numbers ending in 0 = $1 \times 3 \times 2 \times 1 = 6$.

Case 2: The units digit is 5.

Units place: 1 choice (5).

Thousands place: 2 choices (2 or 3, as 0 cannot be in the thousands place and 5 is used).

Hundreds place: 2 choices (0 and the remaining digit from {2, 3}).

Tens place: 1 choice (the last remaining digit).

Number of four-digit numbers ending in 5 = $1 \times 2 \times 2 \times 1 = 4$.

Total number of favorable outcomes (numbers divisible by 5) = (Numbers ending in 0) + (Numbers ending in 5) = $6 + 4 = 10$.


The probability that a randomly formed number is divisible by 5 is the ratio of the number of favorable outcomes to the total number of possible outcomes.

Probability = $\frac{\text{Number of numbers divisible by 5}}{\text{Total number of 4-digit numbers}}$

Probability = $\frac{10}{18}$

Simplify the fraction:

Probability = $\frac{\cancel{10}^{5}}{\cancel{18}_{9}} = \frac{5}{9}$


The probability that the number is divisible by 5 is $\frac{5}{9}$.

This corresponds to option (D).

Question 23. If A and B are mutually exclusive events, then

(A) P (A) ≤ P ($\overline{B}$)

(B) P (A) ≥ P ($\overline{B}$)

(C) P (A) < P ($\overline{B}$)

(D) none of these

Answer:

Given:

A and B are mutually exclusive events.


To Find:

The correct relationship between $P(A)$ and $P(\overline{B})$.


Solution:

Since A and B are mutually exclusive events, their intersection is the empty set.

$A \cap B = \emptyset$

This means that if event A occurs, event B cannot occur. If event B does not occur, then its complement $\overline{B}$ must occur.

Consider an outcome $\omega$ in the sample space.

If $\omega \in A$, then since A and B are mutually exclusive, $\omega \notin B$.

If $\omega \notin B$, then $\omega \in \overline{B}$.

Thus, any outcome that is in A is also in $\overline{B}$. This implies that event A is a subset of event $\overline{B}$.

$A \subseteq \overline{B}$

For any two events $E_1$ and $E_2$ in the same sample space, if $E_1 \subseteq E_2$, then the probability of $E_1$ is less than or equal to the probability of $E_2$.

Since $A \subseteq \overline{B}$, we have:

$P(A) \leq P(\overline{B})$


The correct relationship is $P(A) \leq P(\overline{B})$.

This corresponds to option (A).

Question 24. If P (A ∪ B) = P (A ∩ B) for any two events A and B, then

(A) P (A) = P (B)

(B) P (A) > P (B)

(C) P (A) < P (B)

(D) none of these

Answer:

Given:

For two events A and B, the probability of their union is equal to the probability of their intersection: $P(A \cup B) = P(A \cap B)$.


To Determine:

Which of the given statements must be true based on the given condition.


Solution:

We use the Addition Law of Probability for any two events A and B:

$P(A \cup B) = P(A) + P(B) - P(A \cap B)$

Given that $P(A \cup B) = P(A \cap B)$, we can substitute this into the addition law equation:

$P(A \cap B) = P(A) + P(B) - P(A \cap B)$

Add $P(A \cap B)$ to both sides of the equation:

$2 P(A \cap B) = P(A) + P(B)$

We know that the probability of the intersection of two events is always less than or equal to the probability of each individual event:

$P(A \cap B) \leq P(A)$

$P(A \cap B) \leq P(B)$

From $2 P(A \cap B) = P(A) + P(B)$ and $P(A \cap B) \leq P(A)$, we get:

$P(A) + P(B) \leq 2 P(A)$

$P(B) \leq P(A)$

Similarly, from $2 P(A \cap B) = P(A) + P(B)$ and $P(A \cap B) \leq P(B)$, we get:

$P(A) + P(B) \leq 2 P(B)$

$P(A) \leq P(B)$

Since both $P(B) \leq P(A)$ and $P(A) \leq P(B)$ must be true, we conclude that:

$P(A) = P(B)$

Substitute $P(B) = P(A)$ back into the equation $2 P(A \cap B) = P(A) + P(B)$:

$2 P(A \cap B) = P(A) + P(A)$

$2 P(A \cap B) = 2 P(A)$

$P(A \cap B) = P(A)$

Since $P(A) = P(B)$ and $P(A) = P(A \cap B)$, we also have $P(B) = P(A \cap B)$.

The given condition $P(A \cup B) = P(A \cap B)$ is equivalent to $P(A) = P(B) = P(A \cap B)$.

This implies that the probability of event A is equal to the probability of event B, and all of this probability is contained within their intersection. This means the events A and B are probabilistically equivalent (they differ only by sets of probability zero), i.e., $P(A \Delta B) = 0$.

Now let's examine the given options. We need to determine which relationship between $P(A)$ and $P(\overline{B})$ must be true given $P(A) = P(B)$.

The probability of the complement of B is $P(\overline{B}) = 1 - P(B)$. Since $P(B) = P(A)$, we have $P(\overline{B}) = 1 - P(A)$.

Let $p = P(A)$. Then $P(B) = p$ and $P(\overline{B}) = 1 - p$. The condition $P(A) = P(B) = P(A \cap B)$ is satisfied if $A$ and $B$ are the same event with probability $p$, where $p$ can be any value such that $0 \leq p \leq 1$.

Now let's check the options by comparing $p$ and $1-p$:

(A) $P(A) \leq P(\overline{B}) \implies p \leq 1 - p \implies 2p \leq 1 \implies p \leq 0.5$. This is true if $P(A) \leq 0.5$, but not necessarily true for all $P(A)$ (e.g., if $P(A)=0.6$).

(B) $P(A) \geq P(\overline{B}) \implies p \geq 1 - p \implies 2p \geq 1 \implies p \geq 0.5$. This is true if $P(A) \geq 0.5$, but not necessarily true for all $P(A)$ (e.g., if $P(A)=0.4$).

(C) $P(A) < P(\overline{B}) \implies p < 1 - p \implies 2p < 1 \implies p < 0.5$. This is true if $P(A) < 0.5$, but not necessarily true for all $P(A)$ (e.g., if $P(A)=0.5$ or $P(A)=0.6$).

Since the condition $P(A \cup B) = P(A \cap B)$ implies $P(A) = P(B)$, but does not restrict the value of $P(A)$ to be either $\leq 0.5$ or $\geq 0.5$ or $< 0.5$, none of the inequalities in options (A), (B), or (C) must necessarily be true.

Therefore, the correct answer is (D).


The final answer is $\boxed{D}$.

Question 25. 6 boys and 6 girls sit in a row at random. The probability that all the girls sit together is

(A) $\frac{1}{432}$

(B) $\frac{12}{431}$

(C) $\frac{1}{132}$

(D) none of these

Answer:

Given:

6 boys and 6 girls are to sit in a row.

Total number of people = 6 boys + 6 girls = 12.


To Find:

The probability that all the girls sit together.


Solution:

Let's assume all boys are distinct and all girls are distinct for counting permutations.

Total Number of Arrangements:

The total number of ways to arrange 12 distinct people in a row is given by the factorial of the total number of people.

Total number of arrangements $= 12!$


Favorable Arrangements (All girls sit together):

To count the arrangements where all 6 girls sit together, we treat the group of 6 girls as a single unit or block.

Now, we are arranging the 6 boys and this single unit of 6 girls. This gives us a total of $6 + 1 = 7$ entities to arrange.

The number of ways to arrange these 7 distinct entities is $7!$.

Within the unit of 6 girls, the girls themselves can be arranged in $6!$ ways.

The total number of favorable arrangements where all girls sit together is the product of the number of ways to arrange the 7 entities and the number of ways to arrange the girls within their unit.

Number of favorable arrangements $= (\text{Number of ways to arrange 7 entities}) \times (\text{Number of ways to arrange 6 girls within unit})$

Number of favorable arrangements $= 7! \times 6!$


Probability:

The probability that all the girls sit together is the ratio of the number of favorable arrangements to the total number of arrangements.

Probability $= \frac{\text{Number of favorable arrangements}}{\text{Total number of arrangements}}$

Probability $= \frac{7! \times 6!}{12!}$

We can simplify this expression:

Recall that $12! = 12 \times 11 \times 10 \times 9 \times 8 \times 7!$

Probability $= \frac{7! \times 6!}{12 \times 11 \times 10 \times 9 \times 8 \times 7!}$

Cancel out $7!$ from the numerator and the denominator:

Probability $= \frac{6!}{12 \times 11 \times 10 \times 9 \times 8}$

Substitute the value of $6! = 720$:

Probability $= \frac{720}{12 \times 11 \times 10 \times 9 \times 8}$

Calculate the product in the denominator: $12 \times 11 \times 10 \times 9 \times 8 = 95,040$

Probability $= \frac{720}{95040}$

Simplify the fraction:

Probability $= \frac{\cancel{720}^{72}}{\cancel{95040}_{9504}}$

Now simplify $\frac{72}{9504}$ by dividing both numerator and denominator by 72:

$72 \div 72 = 1$

$9504 \div 72 = 132$

Probability $= \frac{1}{132}$


The probability that all the girls sit together is $\frac{1}{132}$.

This corresponds to option (C).

Question 26. A single letter is selected at random from the word ‘PROBABILITY’. The probability that it is a vowel is

(A) $\frac{1}{3}$

(B) $\frac{4}{11}$

(C) $\frac{2}{11}$

(D) $\frac{3}{11}$

Answer:

Given:

A single letter is selected at random from the word ‘PROBABILITY’.


To Find:

The probability that the selected letter is a vowel.


Solution:

First, list the letters in the word ‘PROBABILITY’ and count the total number of letters.

The letters are P, R, O, B, A, B, I, L, I, T, Y.

Total number of letters in the word = 11.

Now, identify the vowels in the word. The vowels are A, E, I, O, U.

In the word ‘PROBABILITY’, the vowels are O, A, I, I.

Count the number of vowel letters in the word.

Number of vowel letters = 4 (O, A, I, I).

The probability of selecting a vowel is the ratio of the number of vowel letters to the total number of letters.

Probability (vowel) $= \frac{\text{Number of vowel letters}}{\text{Total number of letters}}$

Probability (vowel) $= \frac{4}{11}$


The probability that the selected letter is a vowel is $\frac{4}{11}$.

This corresponds to option (B).

Question 27. If the probabilities for A to fail in an examination is 0.2 and that for B is 0.3, then the probability that either A or B fails is

(A) > . 5

(B) .5

(C) ≤ .5

(D) 0

Answer:

Given:

Let A be the event that A fails in the examination, so $P(A) = 0.2$.

Let B be the event that B fails in the examination, so $P(B) = 0.3$.


To Find:

The probability that either A or B fails, which is $P(A \cup B)$.


Solution:

The general formula for the probability of the union of two events A and B is:

$P(A \cup B) = P(A) + P(B) - P(A \cap B)$

We are given $P(A) = 0.2$ and $P(B) = 0.3$. Substitute these values into the formula:

$P(A \cup B) = 0.2 + 0.3 - P(A \cap B)$

$P(A \cup B) = 0.5 - P(A \cap B)$

The term $P(A \cap B)$ represents the probability that both A and B fail. We know that the probability of any event must be non-negative.

$P(A \cap B) \geq 0$

Substitute this inequality into the expression for $P(A \cup B)$:

$P(A \cup B) = 0.5 - P(A \cap B) \leq 0.5 - 0$

$P(A \cup B) \leq 0.5$

The maximum value of $P(A \cup B)$ is 0.5, which occurs when $P(A \cap B) = 0$ (i.e., if the events of A and B failing are mutually exclusive).

The minimum value of $P(A \cap B)$ cannot exceed the probabilities of A failing or B failing individually. That is, $P(A \cap B) \leq P(A) = 0.2$ and $P(A \cap B) \leq P(B) = 0.3$. So, $P(A \cap B) \leq \min(0.2, 0.3) = 0.2$.

Using the maximum possible value for $P(A \cap B)$: $P(A \cup B) = 0.5 - P(A \cap B) \geq 0.5 - 0.2 = 0.3$.

So, the probability $P(A \cup B)$ is always between 0.3 and 0.5 (inclusive): $0.3 \leq P(A \cup B) \leq 0.5$.

Based on the inequality $P(A \cup B) \leq 0.5$, we can evaluate the given options:

(A) $P(A \cup B) > 0.5$: This is not always true. For example, if $P(A \cap B) > 0$, then $P(A \cup B) < 0.5$.

(B) $P(A \cup B) = 0.5$: This is only true in the specific case where $P(A \cap B) = 0$ (mutually exclusive failure events), but not necessarily for all cases.

(C) $P(A \cup B) \leq 0.5$: This is always true because $P(A \cap B)$ cannot be negative.

(D) $P(A \cup B) = 0$: This is only true if $P(A) = 0$ and $P(B) = 0$, which is not the case here.


The probability that either A or B fails is always less than or equal to 0.5.

This corresponds to option (C).

Question 28. The probability that at least one of the events A and B occurs is 0.6. If A and B occur simultaneously with probability 0.2, then P ($\overline{A}$) + P ($\overline{B}$) is

(A) 0.4

(B) 0.8

(C) 1.2

(D) 1.6

Answer:

Given:

The probability that at least one of the events A and B occurs is 0.6. This means the probability of the union of A and B is 0.6.

$P(A \cup B) = 0.6$

A and B occur simultaneously with probability 0.2. This means the probability of the intersection of A and B is 0.2.

$P(A \cap B) = 0.2$


To Find:

The value of $P(\overline{A}) + P(\overline{B})$.


Solution:

We know the relationship between the probability of an event and the probability of its complement:

$P(\overline{A}) = 1 - P(A)$

$P(\overline{B}) = 1 - P(B)$

We want to find $P(\overline{A}) + P(\overline{B}) = (1 - P(A)) + (1 - P(B)) = 2 - (P(A) + P(B))$.

To find $P(A) + P(B)$, we can use the Addition Law of Probability:

$P(A \cup B) = P(A) + P(B) - P(A \cap B)$

Substitute the given values:

$0.6 = P(A) + P(B) - 0.2$

Add 0.2 to both sides of the equation to find the sum $P(A) + P(B)$:

$0.6 + 0.2 = P(A) + P(B)$

$P(A) + P(B) = 0.8$

Now substitute this sum into the expression for $P(\overline{A}) + P(\overline{B})$:

$P(\overline{A}) + P(\overline{B}) = 2 - (P(A) + P(B))$

$P(\overline{A}) + P(\overline{B}) = 2 - 0.8$

$P(\overline{A}) + P(\overline{B}) = 1.2$


The value of $P(\overline{A}) + P(\overline{B})$ is 1.2.

This corresponds to option (C).

Question 29. If M and N are any two events, the probability that at least one of them occurs is

(A) P (M) + P (N) – 2 P (M ∩ N)

(B) P (M) + P (N) – P (M ∩ N)

(C) P (M) + P (N) + P (M ∩ N)

(D) P (M) + P (N) + 2P (M ∩ N)

Answer:

Given:

Two events M and N.


To Find:

The probability that at least one of them occurs.


Solution:

The phrase "at least one of the events M and N occurs" corresponds to the union of the two events, denoted as $M \cup N$.

The probability of the union of any two events M and N is given by the Addition Law of Probability.

The Addition Law states that:

$P(M \cup N) = P(M) + P(N) - P(M \cap N)$

where $P(M \cap N)$ is the probability that both events M and N occur simultaneously (the intersection of M and N).

Comparing this formula with the given options, we find that it matches option (B).


The probability that at least one of the events M and N occurs is $P(M) + P(N) - P(M \cap N)$.

This corresponds to option (B).

Question 30 to 36 (True or False)

State whether the statements are True or False in each of the Exercises 30 to 36.

Question 30. The probability that a person visiting a zoo will see the giraffee is 0.72, the probability that he will see the bears is 0.84 and the probability that he will see both is 0.52.

Answer:

Given:

Let G be the event that the person sees the giraffes.

Let B be the event that the person sees the bears.

We are given:

$P(G) = 0.72$

$P(B) = 0.84$

$P(G \cap B) = 0.52$


To Determine:

Whether these probabilities are consistent with the rules of probability.


Solution:

For any two events G and B, the probability of their union (seeing either giraffes or bears or both) is given by the Addition Law of Probability:

$P(G \cup B) = P(G) + P(B) - P(G \cap B)$

Substitute the given probabilities into the formula:

$P(G \cup B) = 0.72 + 0.84 - 0.52$

$P(G \cup B) = 1.56 - 0.52$

$P(G \cup B) = 1.04$

According to the axioms of probability, the probability of any event must be between 0 and 1 (inclusive).

That is, for any event E, $0 \leq P(E) \leq 1$.

In this case, $P(G \cup B) = 1.04$, which is greater than 1.

Since the probability of the union of the two events exceeds 1, the given probabilities are inconsistent and cannot represent probabilities of real-world events.


Therefore, the statement is False.

Question 31. The probability that a student will pass his examination is 0.73, the probability of the student getting a compartment is 0.13, and the probability that the student will either pass or get compartment is 0.96.

Answer:

Given:

Let P be the event that the student passes the examination.

$P(P) = 0.73$

Let C be the event that the student gets a compartment.

$P(C) = 0.13$

The probability that the student will either pass or get compartment is $P(P \cup C) = 0.96$.


To Determine:

Whether the given probabilities are consistent.


Solution:

In the context of a single examination result, the event of passing and the event of getting a compartment are mutually exclusive. A student cannot simultaneously pass the examination and get a compartment in the same subject(s) or examination attempt.

For mutually exclusive events P and C, the probability of their union is given by the Addition Law of Probability:

$P(P \cup C) = P(P) + P(C)$

Substitute the given probabilities for $P(P)$ and $P(C)$ into this formula:

$P(P \cup C) = 0.73 + 0.13$

$P(P \cup C) = 0.86$

The calculated probability of the student either passing or getting a compartment is 0.86.

However, the problem states that the probability of the student either passing or getting compartment is 0.96.

$0.86 \neq 0.96$

Since the given probability of the union (0.96) does not match the probability calculated using the Addition Law for mutually exclusive events (0.86), the given probabilities are inconsistent with the rules of probability for mutually exclusive events.


Therefore, the statement is False.

Question 32. The probabilities that a typist will make 0, 1, 2, 3, 4, 5 or more mistakes in typing a report are, respectively, 0.12, 0.25, 0.36, 0.14, 0.08, 0.11.

Answer:

Given:

The probabilities of the number of mistakes made by a typist are given as:

$P(\text{0 mistakes}) = 0.12$

$P(\text{1 mistake}) = 0.25$

$P(\text{2 mistakes}) = 0.36$

$P(\text{3 mistakes}) = 0.14$

$P(\text{4 mistakes}) = 0.08$

$P(\text{5 or more mistakes}) = 0.11$


To Determine:

Whether these probabilities are consistent.


Solution:

The given events (making 0, 1, 2, 3, 4, or 5 or more mistakes) form a set of mutually exclusive and exhaustive outcomes for the number of mistakes made by the typist. This means that the sum of the probabilities of these events must be equal to 1.

Sum of probabilities $= P(\text{0}) + P(\text{1}) + P(\text{2}) + P(\text{3}) + P(\text{4}) + P(\text{5 or more})$

Sum of probabilities $= 0.12 + 0.25 + 0.36 + 0.14 + 0.08 + 0.11$

Let's add these values:

$0.12 + 0.25 = 0.37$

$0.37 + 0.36 = 0.73$

$0.73 + 0.14 = 0.87$

$0.87 + 0.08 = 0.95$

$0.95 + 0.11 = 1.06$

The sum of the given probabilities is 1.06.

According to the axioms of probability, the sum of the probabilities of all elementary outcomes in a sample space must be exactly 1.

Since the sum of the probabilities in this case is 1.06, which is greater than 1, the given probabilities are inconsistent.


Therefore, the statement is False.

Question 33. If A and B are two candidates seeking admission in an engineering College. The probability that A is selected is .5 and the probability that both A and B are selected is at most .3. Is it possible that the probability of B getting selected is 0.7?

Answer:

Given:

Let A be the event that candidate A is selected.

Let B be the event that candidate B is selected.

We are given:

$P(A) = 0.5$

$P(A \cap B) \leq 0.3$

We want to determine if it is possible that $P(B) = 0.7$.


To Determine:

Whether the statement "it is possible that the probability of B getting selected is 0.7" is True or False, given the other conditions.


Solution:

Let's assume that $P(B) = 0.7$. We need to check if this assumption is consistent with the given conditions $P(A) = 0.5$ and $P(A \cap B) \leq 0.3$, along with the fundamental axioms of probability ($0 \leq P(E) \leq 1$ for any event E, and $P(E_1 \cup E_2) = P(E_1) + P(E_2) - P(E_1 \cap E_2)$).

For any two events A and B, the probability of their intersection is always less than or equal to the probability of each individual event:

$P(A \cap B) \leq P(A)$

$P(A \cap B) \leq P(B)$

If we assume $P(A) = 0.5$ and $P(B) = 0.7$, these inequalities become:

$P(A \cap B) \leq 0.5$

$P(A \cap B) \leq 0.7$

Combining these, we must have $P(A \cap B) \leq \min(0.5, 0.7) = 0.5$.

We are also given the condition $P(A \cap B) \leq 0.3$.

For the assumption $P(B) = 0.7$ to be possible, there must exist a value for $P(A \cap B)$ that satisfies both $P(A \cap B) \leq 0.5$ and $P(A \cap B) \leq 0.3$. The combined upper bound is $P(A \cap B) \leq \min(0.5, 0.3) = 0.3$. So, any value of $P(A \cap B)$ between 0 and 0.3 (inclusive) would satisfy these conditions from the individual probabilities.

Now let's consider the Addition Law of Probability:

$P(A \cup B) = P(A) + P(B) - P(A \cap B)$

Using the assumed $P(A) = 0.5$ and $P(B) = 0.7$:

$P(A \cup B) = 0.5 + 0.7 - P(A \cap B) = 1.2 - P(A \cap B)$

We know that the probability of the union of any two events must be between 0 and 1 (inclusive):

$0 \leq P(A \cup B) \leq 1$

Substituting the expression for $P(A \cup B)$:

$0 \leq 1.2 - P(A \cap B) \leq 1$

From the right side of the inequality: $1.2 - P(A \cap B) \leq 1 \implies 1.2 - 1 \leq P(A \cap B) \implies 0.2 \leq P(A \cap B)$.

From the left side of the inequality: $0 \leq 1.2 - P(A \cap B) \implies P(A \cap B) \leq 1.2$. This is always true since $P(A \cap B) \leq P(A) = 0.5$ (and $0.5 \leq 1.2$).

So, if $P(A) = 0.5$ and $P(B) = 0.7$, the probability of the intersection must satisfy $P(A \cap B) \geq 0.2$.

Combining the constraints on $P(A \cap B)$: We are given $P(A \cap B) \leq 0.3$, and we derived $P(A \cap B) \geq 0.2$ from the assumption $P(B) = 0.7$ and $P(A) = 0.5$.

Thus, for $P(B) = 0.7$ to be possible, there must exist a value of $P(A \cap B)$ such that $0.2 \leq P(A \cap B) \leq 0.3$.

Since the interval $[0.2, 0.3]$ is a valid range for a probability (all values are between 0 and 1), and it satisfies both the condition from $P(A), P(B)$ and the separately given condition $P(A \cap B) \leq 0.3$, it is possible for $P(B)$ to be 0.7 under the given circumstances.

For instance, if $P(A)=0.5$, $P(B)=0.7$, and $P(A \cap B)=0.25$, then all conditions are met:

$P(A)=0.5$ (given)

$P(B)=0.7$ (assumed possible)

$P(A \cap B)=0.25 \leq 0.3$ (given condition satisfied)

$P(A \cap B)=0.25 \leq \min(P(A), P(B)) = \min(0.5, 0.7) = 0.5$ (fundamental rule satisfied)

$P(A \cup B) = P(A) + P(B) - P(A \cap B) = 0.5 + 0.7 - 0.25 = 1.2 - 0.25 = 0.95$. Since $0 \leq 0.95 \leq 1$, the probability of the union is valid.

Since we found a possible scenario where all conditions are met, the statement is True.


Therefore, the statement is True.

Question 34. The probability of intersection of two events A and B is always less than or equal to those favourable to the event A.

Answer:

Given Statement:

The probability of intersection of two events A and B is always less than or equal to those favourable to the event A.


To Determine:

Whether the given statement is True or False.


Solution:

Let A and B be any two events in a sample space S.

The intersection of events A and B is the event $A \cap B$, which consists of all outcomes that are common to both event A and event B.

"Those favourable to the event A" refers to the outcomes that constitute event A itself. The probability of event A is $P(A)$.

The statement is claiming that $P(A \cap B) \leq P(A)$.

By definition, the set of outcomes in the intersection $A \cap B$ is a subset of the set of outcomes in A.

$A \cap B \subseteq A$

A fundamental property of probability states that if an event $E_1$ is a subset of another event $E_2$ (i.e., $E_1 \subseteq E_2$), then the probability of $E_1$ is less than or equal to the probability of $E_2$.

Since $A \cap B \subseteq A$, it follows that $P(A \cap B) \leq P(A)$.

This property holds true for any two events A and B.


Therefore, the statement "The probability of intersection of two events A and B is always less than or equal to those favourable to the event A" is True.

Question 35. The probability of an occurrence of event A is .7 and that of the occurrence of event B is .3 and the probability of occurrence of both is .4.

Answer:

Given:

Let A and B be two events.

We are given the following probabilities:

$P(A) = 0.7$

$P(B) = 0.3$

$P(A \cap B) = 0.4$


To Determine:

Whether the given probabilities are consistent with the rules of probability.


Solution:

For any two events A and B, the probability of their intersection ($A \cap B$) must be less than or equal to the probability of each individual event ($A$ and $B$).

That is, we must have:

$P(A \cap B) \leq P(A)$

$P(A \cap B) \leq P(B)$

Let's check the first condition with the given values:

$0.4 \leq 0.7$

This condition is satisfied.

Now, let's check the second condition:

$0.4 \leq 0.3$

This condition is NOT satisfied. The probability of both events occurring (0.4) cannot be greater than the probability of event B occurring alone (0.3), because the outcomes where both A and B occur are a subset of the outcomes where B occurs.

Since one of the fundamental rules of probability is violated, the given probabilities are inconsistent.


Therefore, the statement is False.

Question 36. The sum of probabilities of two students getting distinction in their final examinations is 1.2.

Answer:

Given Statement:

The sum of probabilities of two students getting distinction in their final examinations is 1.2.


To Determine:

Whether the given statement is True or False.


Solution:

Let $S_1$ be the event that the first student gets distinction in their final examinations.

Let $S_2$ be the event that the second student gets distinction in their final examinations.

The statement claims that the sum of the probabilities of these two events is 1.2, i.e., $P(S_1) + P(S_2) = 1.2$.

According to the axioms of probability, the probability of any event must be a number between 0 and 1, inclusive.

$0 \leq P(S_1) \leq 1$

$0 \leq P(S_2) \leq 1$

The statement asks if it is possible for the sum of two valid probabilities to be 1.2. Let's see if we can find values for $P(S_1)$ and $P(S_2)$ that satisfy both the probability axioms and the given sum.

For example, let $P(S_1) = 0.6$ and $P(S_2) = 0.6$. Both these probabilities are valid since $0 \leq 0.6 \leq 1$.

Their sum is $P(S_1) + P(S_2) = 0.6 + 0.6 = 1.2$.

Another example: Let $P(S_1) = 0.5$ and $P(S_2) = 0.7$. Both are valid probabilities, and their sum is $0.5 + 0.7 = 1.2$.

Since it is possible to have two distinct events whose individual probabilities sum up to 1.2 (as long as each individual probability is less than or equal to 1), the statement that the sum *is* 1.2 in a specific scenario is possible within the rules of probability.

Note that this is different from the probability of the union of the two events, $P(S_1 \cup S_2)$, which represents the probability that at least one of the students gets distinction. The probability of the union must always be less than or equal to 1, i.e., $P(S_1 \cup S_2) \leq 1$. However, the statement is about the sum of the individual probabilities, not the probability of the union.


Since we can find valid probabilities $P(S_1)$ and $P(S_2)$ whose sum is 1.2, the statement that the sum of probabilities is 1.2 is possible.

Therefore, the statement is True.

Question 37 to 41 (Fill in the Blanks)

Fill in the blanks in the Exercises 37 to 41.

Question 37. The probability that the home team will win an upcoming football game is 0.77, the probability that it will tie the game is 0.08, and the probability that it will lose the game is _____.

Answer:

Given:

Let W be the event that the home team wins.

$P(W) = 0.77$

Let T be the event that the home team ties.

$P(T) = 0.08$


To Find:

The probability that the home team will lose the game.


Solution:

In a football game, there are three possible mutually exclusive and exhaustive outcomes for the home team: Win, Tie, or Lose.

Let L be the event that the home team loses.

The sum of the probabilities of all mutually exclusive and exhaustive events in a sample space must equal 1.

$P(W) + P(T) + P(L) = 1$

Substitute the given probabilities into the equation:

$0.77 + 0.08 + P(L) = 1$

Add the known probabilities:

$0.85 + P(L) = 1$

Subtract 0.85 from 1 to find $P(L)$:

$P(L) = 1 - 0.85$

$P(L) = 0.15$


The probability that the home team will lose the game is 0.15.

The completed statement is: The probability that the home team will win an upcoming football game is 0.77, the probability that it will tie the game is 0.08, and the probability that it will lose the game is 0.15.

Question 38. If e1, e2, e3, e4 are the four elementary outcomes in a sample space and P(e1) = .1, P(e2) = .5, P (e3) = .1, then the probability of e4 is ______.

Answer:

Given:

The sample space consists of four elementary outcomes: $e_1, e_2, e_3, e_4$.

The probabilities of three of these outcomes are given:

$P(e_1) = 0.1$

$P(e_2) = 0.5$

$P(e_3) = 0.1$


To Find:

The probability of the elementary outcome $e_4$, $P(e_4)$.


Solution:

A fundamental property of probabilities for elementary outcomes is that the sum of the probabilities of all distinct elementary outcomes in a sample space must equal 1.

For the given sample space $\{e_1, e_2, e_3, e_4\}$, the sum of probabilities is:

$P(e_1) + P(e_2) + P(e_3) + P(e_4) = 1$

Substitute the given probabilities into this equation:

$0.1 + 0.5 + 0.1 + P(e_4) = 1$

Sum the known probabilities:

$0.7 + P(e_4) = 1$

Solve for $P(e_4)$ by subtracting 0.7 from 1:

$P(e_4) = 1 - 0.7$

$P(e_4) = 0.3$


The probability of $e_4$ is 0.3.

The completed statement is: If e1, e2, e3, e4 are the four elementary outcomes in a sample space and P(e1) = .1, P(e2) = .5, P (e3) = .1, then the probability of e4 is 0.3.

Question 39. Let S = {1, 2, 3, 4, 5, 6} and E = {1, 3, 5}, then $\overline{E}$ is _________.

Answer:

Given:

Sample space $S = \{1, 2, 3, 4, 5, 6\}$.

Event $E = \{1, 3, 5\}$.


To Find:

The complement of event E, denoted as $\overline{E}$ or $E'$.


Solution:

The complement of an event E with respect to the sample space S is the set of all outcomes in S that are not in E.

$\overline{E} = \{x \in S \mid x \notin E\}$

The elements in S are {1, 2, 3, 4, 5, 6}.

The elements in E are {1, 3, 5}.

To find the elements in $\overline{E}$, we remove the elements of E from S:

$S \setminus E = \{1, 2, 3, 4, 5, 6\} \setminus \{1, 3, 5\} = \{2, 4, 6\}$

So, the complement of E is $\overline{E} = \{2, 4, 6\}$.


The completed statement is: Let S = {1, 2, 3, 4, 5, 6} and E = {1, 3, 5}, then $\overline{E}$ is {2, 4, 6}.

Question 40. If A and B are two events associated with a random experiment such that P ( A) = 0.3, P (B) = 0.2 and P (A ∩ B) = 0.1, then the value of P (A ∩ $\overline{B}$) is _______.

Answer:

Given:

A and B are two events.

$P(A) = 0.3$

$P(B) = 0.2$

$P(A \cap B) = 0.1$


To Find:

The value of $P(A \cap \overline{B})$.


Solution:

The event $A \cap \overline{B}$ represents the outcomes that are in event A but not in event B. This is sometimes referred to as the event "A occurs, but B does not occur".

The event A can be partitioned into two disjoint events: the part that is also in B ($A \cap B$) and the part that is not in B ($A \cap \overline{B}$).

$A = (A \cap B) \cup (A \cap \overline{B})$

Since the events $(A \cap B)$ and $(A \cap \overline{B})$ are mutually exclusive (they have no outcomes in common), the probability of their union is the sum of their probabilities:

$P(A) = P(A \cap B) + P(A \cap \overline{B})$

We are given $P(A)$ and $P(A \cap B)$, and we want to find $P(A \cap \overline{B})$. Rearrange the formula to solve for $P(A \cap \overline{B})$:

$P(A \cap \overline{B}) = P(A) - P(A \cap B)$

Substitute the given values:

$P(A \cap \overline{B}) = 0.3 - 0.1$

$P(A \cap \overline{B}) = 0.2$


The value of $P(A \cap \overline{B})$ is 0.2.

The completed statement is: If A and B are two events associated with a random experiment such that P ( A) = 0.3, P (B) = 0.2 and P (A ∩ B) = 0.1, then the value of P (A ∩ $\overline{B}$) is 0.2.

Question 41. The probability of happening of an event A is 0.5 and that of B is 0.3. If A and B are mutually exclusive events, then the probability of neither A nor B is ________.

Answer:

Given:

$P(A) = 0.5$

$P(B) = 0.3$

A and B are mutually exclusive events.


To Find:

The probability of neither A nor B.


Solution:

The event "neither A nor B" means that event A does not occur and event B does not occur. This corresponds to the intersection of the complements of A and B, which is $\overline{A} \cap \overline{B}$.

According to De Morgan's Law, the complement of the union of two events is equal to the intersection of their complements:

$\overline{A} \cap \overline{B} = (A \cup B)'$

So, $P(\text{neither A nor B}) = P(\overline{A} \cap \overline{B}) = P((A \cup B)')$.

Using the complement rule, the probability of the complement of an event is 1 minus the probability of the event:

$P((A \cup B)') = 1 - P(A \cup B)$

We need to find $P(A \cup B)$. Since A and B are mutually exclusive events, the probability of their union is the sum of their individual probabilities:

$P(A \cup B) = P(A) + P(B)$

Substitute the given probabilities:

$P(A \cup B) = 0.5 + 0.3$

$P(A \cup B) = 0.8$

Now, substitute this value back into the expression for the probability of neither A nor B:

$P(\text{neither A nor B}) = 1 - P(A \cup B)$

$P(\text{neither A nor B}) = 1 - 0.8$

$P(\text{neither A nor B}) = 0.2$


The probability of neither A nor B is 0.2.

The completed statement is: The probability of happening of an event A is 0.5 and that of B is 0.3. If A and B are mutually exclusive events, then the probability of neither A nor B is 0.2.

Question 42 to 43 (Match the Following)

Question 42. Match the proposed probability under Column C1 with the appropriate written description under column C2 :

$C_1$ (Probability)

(a) 0.95

(b) 0.02

(c) – 0.3

(d) 0.5

(e) 0

$C_2$ (Written Description)

(i) An incorrect assignment

(ii) No chance of happening

(iii) As much chance of happening as not.

(iv) Very likely to happen

(v) Very little chance of happening

Answer:

Solution:


Let's match each probability in Column C1 with the most appropriate description in Column C2 based on the interpretation of probability values.


(a) 0.95: This is a high probability, close to 1. It signifies that the event is very likely to occur.

This matches description (iv) Very likely to happen.


(b) 0.02: This is a very low positive probability, close to 0. It signifies that the event has a very small chance of occurring.

This matches description (v) Very little chance of happening.


(c) – 0.3: Probabilities must always be non-negative. A negative value is not a valid probability.

This matches description (i) An incorrect assignment.


(d) 0.5: This probability represents an equal chance of the event happening or not happening (50% chance).

This matches description (iii) As much chance of happening as not.


(e) 0: This probability signifies that the event is impossible; it has no chance of occurring.

This matches description (ii) No chance of happening.


The matches are:

(a) $\to$ (iv)

(b) $\to$ (v)

(c) $\to$ (i)

(d) $\to$ (iii)

(e) $\to$ (ii)

Question 43. Match the following

(a) If $E_1$ and $E_2$ are the two mutually exclusive events

(b) If $E_1$ and $E_2$ are the mutually exclusive and exhaustive events

(c) If $E_1$ and $E_2$ have common outcomes, then

(d) If $E_1$ and $E_2$ are two events such that $E_1 \subset E_2$

(i) $E_1 \cap E_2 = E_1$

(ii) $(E_1 – E_2) \cup (E_1 \cap E_2) = E_1$

(iii) $E_1 \cap E_2 = \phi$, $E_1 \cup E_2 = S$

(iv) $E_1 \cap E_2 = \phi$

Answer:

Solution:


Let's match the descriptions in the left column with the corresponding properties in the right column.


(a) If $E_1$ and $E_2$ are the two mutually exclusive events:

Mutually exclusive events are events that cannot occur at the same time. This means their intersection is the empty set ($\phi$).

This corresponds to property (iv) $E_1 \cap E_2 = \phi$.


(b) If $E_1$ and $E_2$ are the mutually exclusive and exhaustive events:

Mutually exclusive means $E_1 \cap E_2 = \phi$.

Exhaustive events are events whose union covers the entire sample space S, i.e., $E_1 \cup E_2 = S$.

This corresponds to property (iii) $E_1 \cap E_2 = \phi$, $E_1 \cup E_2 = S$.


(c) If $E_1$ and $E_2$ have common outcomes, then:

Having common outcomes means their intersection is not empty, $E_1 \cap E_2 \neq \phi$. The property listed in option (ii), $(E_1 – E_2) \cup (E_1 \cap E_2) = E_1$, is a general set identity that is always true for any two sets $E_1$ and $E_2$, regardless of whether they have common outcomes or not. The set $E_1$ can be partitioned into the part not in $E_2$ ($E_1 - E_2$ or $E_1 \setminus E_2$) and the part in $E_2$ ($E_1 \cap E_2$). The union of these disjoint parts is $E_1$. Given the options, this is the only set identity presented.

This corresponds to property (ii) $(E_1 – E_2) \cup (E_1 \cap E_2) = E_1$.


(d) If $E_1$ and $E_2$ are two events such that $E_1 \subset E_2$:

If $E_1$ is a subset of $E_2$, then every outcome in $E_1$ is also an outcome in $E_2$. The intersection of $E_1$ and $E_2$ contains all outcomes that are in both $E_1$ and $E_2$. Since all outcomes of $E_1$ are in $E_2$, the intersection is simply $E_1$.

This corresponds to property (i) $E_1 \cap E_2 = E_1$.


The final matches are:

(a) $\to$ (iv)

(b) $\to$ (iii)

(c) $\to$ (ii)

(d) $\to$ (i)